Adult Health Exam 2 - respiratory, HTN, HF, PVD

Lakukan tugas rumah & ujian kamu dengan baik sekarang menggunakan Quizwiz!

The nurse is providing a community program about lung cancer. What risk factor should the nurse be sure to include when discussing this topic? 1 Age of smoking onset 2 Use of filtered cigarettes 3 Prolonged exposure to nickel 4 Number of cigarettes smoked per day

1 Age of smoking onset is the directly related risk of lung cancer. Use of filtered cigarettes, number of cigarettes per day, and prolonged exposure to nickel may not be directly related. Text Reference - p. 536

When caring for a patient with pertussis, the nurse will prioritize 1 Administering antibiotic therapy 2 Having the patient use an antihistamine at night 3 Teaching the patient how to use a bronchodilator 4 Instructing the patient to use cough suppressants

1 Treatment for pertussis is antibiotics, usually macrolides (erythromycin, azithromycin), to minimize symptoms and prevent spread of the disease. Cough suppressants and antihistamines should not be used, because they are ineffective and may induce coughing episodes. Corticosteroids and bronchodilators are not useful in reducing symptoms. Text Reference - p. 522

The nurse is conducting a health history interview with a patient diagnosed with chronic obstructive pulmonary disease (COPD). Which question is appropriate when assessing the patient's nutritional-metabolic pattern? 1 "Have you lost any weight recently?" 2 "Do you have trouble getting to the toilet?" 3 "Does your breathing wake you up in the night?" 4 "Do you have any pain associated with breathing?"

1 When assessing the effect of COPD on the patient's nutritional-metabolic pattern, the nurse should ask if the patient has experienced any weight loss. Asking about trouble getting to the toilet assesses the effect that COPD has on the patient's elimination patterns. Asking the patient about waking in the middle of the night with breathing issues assesses the patient's sleep-rest pattern. Asking the patient about pain associated with breathing assesses the patient's cognitive-perceptual pattern. Text Reference - p. 483

Which alteration to the suction control system in a wet suction system requires immediate correction? 1 The suction control chamber is open. 2 The suction control chamber shows bubbling. 3 The suction control chamber contains sterile water. 4 The suction tubing is connected to the wall section.

1 = The suction control chamber should not be open; a muffler should be provided to cover the suction control chamber to prevent rapid evaporation of water. The suction control system should show the presence of bubbling; the absence of bubbling indicates that there is no suction. Sterile water is to be added to the suction control chamber to replace water lost from evaporation. The suction tubing should be connected to the wall section after filling the suction control chamber to the ordered suction amount. Text Reference - p. 546

A nurse is caring for a patient with bronchitis. The health care provider has advised arterial blood gas (ABG) analysis. The blood sample can be obtained from which blood vessels? Select all that apply. 1 Radial artery 2 Femoral artery 3 Pulmonary artery 4 Superior vena cava 5 Femoral vein

1, 2 Blood samples for ABG analysis are collected from the radial and femoral arteries, because they carry oxygenated blood. The pulmonary artery carries deoxygenated blood and is used for the measurement of mixed venous blood gas. The superior vena cava is used to measure the central venous pressure (CVP). The femoral vein is not used for ABG analysis, because it carries deoxygenated blood. Text Reference - p. 478

A nurse is caring for a patient with asthma. What instructions should the nurse provide to the patient to help prevent triggers of bronchoconstriction? Select all that apply. 1 Avoid wearing perfumes. 2 Avoid exposure to strong odors. 3 Do not drink cold water. 4 Avoid exposure to cold air. 5 Avoid people with certain infections

1, 2, 4 The different triggers for asthmatic attack include perfumes, strong odors, and cold air. They cause reflex bronchoconstriction, and the patient should avoid them to prevent asthma attacks. Cold water and bacteria do not trigger asthma. Text Reference - p. 481

When the patient has epistaxis upon removal of a nasogastric tube, the nurse places direct pressure on the entire soft lower portion of the nose against the nasal septum for how long?

10 to 15 minutes is indicated for epistaxis.

community acquired PN is associated with an (1) onset with (2 - symptoms), however, the older patient (3)

1: acute 2: fever, chills, productive cough with purulent or bloody sputum, and pleuritic CP 3: may not have classic symptoms - altered LOC or confusion, hypothermia

The nurse is caring for a patient with chronic obstructive pulmonary disease (COPD) and pneumonia who has a prescription for arterial blood gases to be drawn. What is the minimum length of time the nurse should plan to hold pressure on the puncture site? 1: 2 Minutes 2: 5 Minutes 3: 10 Minutes 4: 15 Minutes

2 After obtaining blood for an arterial blood gas measurement, the nurse should hold pressure on the puncture site for five minutes to be sure that bleeding has stopped. An artery is an elastic vessel under much higher pressure than veins, and significant blood loss or hematoma formation could occur if the time is insufficient. Text Reference - p. 491

After assisting the health care provider with a bedside thoracentesis, the nurse should continue to assess the patient for signs and symptoms of what? 1 Bronchospasm 2 Pneumothorax 3 Pulmonary edema 4 Respiratory acidosis

2 Because thoracentesis involves the introduction of a catheter into the pleural space, there is a risk of pneumothorax. Thoracentesis does not carry a significant potential for causing bronchospasm, pulmonary edema, or respiratory acidosis. Text Reference - p. 493

What is the cause of atelectasis? 1 Platelet deficiency 2 Surfactant deficiency 3 Red blood cell deficiency 4 White blood cell deficiency

2 Surfactant is a lipoprotein that lowers the surface tension in the alveoli and reduces the amount of pressure needed to inflate the alveoli, making them less likely to collapse. Therefore, the primary reason that atelectasis occurs is due to a surfactant deficiency. Deficiencies in platelets, red blood cells, and white blood cells do not cause atelectasis. Text Reference - p. 477

Which finding in a patient who is on a chest drainage system requires attention of the primary health care provider immediately? 1 The fluid has clear appearance. 2 The fluid level drained is 120 mL/hr. 3 The water level in suction chamber is 20 cm. 4 The water level in water seal chamber is 2 cm.

2 The normal flow of drainage is below 100 mL/hr. Fluid level drainage of 120 mL/hr indicates excessive drainage or/and increases the risk of further complications, and this requires the immediate attention of the primary health care provider. The presence of yellow or bloody fluid would indicate that the patient may have complications; however, the fluid is clear, indicating no risk of injury or infection. While adjusting the chest drainage tube, the nurse should ensure that the water level in the suction chamber is 20 cm and the water seal chamber is 2 cm. This will ensure that the suction is appropriate.

Which statement made by the patient indicates effective learning about the interventions to be followed to reduce the risk of atelectasis while undergoing chest tube drainage? Select all that apply. 1 "I should change position slowly." 2 "I should cough at regular intervals." 3 "I should perform incentive spirometer." 4 "I should reduce intake of protein in my diet." 5 "I should perform range-of-motion exercises."

2, 3, 5 Coughing and deep breathing exercises will open the pores of Kohn in the lungs and relieve atelectasis. Incentive spirometer will assist in clearing the lungs and will reduce the risk of atelectasis. The nurse instructs the patient to perform range-of-motion exercises because doing so reduces the risk of atelectasis. The nurse instructs the patient to change position slowly if he or she has hypotension. Protein is essential and does not cause any complications for atelectasis. Test-Taking Tip: You should recollect the measures to be followed to prevent the risk of atelectasis in the patient with chest wall injury to answer this question accurately. Text Reference - p. 547

The nurse is assessing a patient who has a closed chest tube drainage system attached to a water-seal chamber. The nurse notes that the tidaling has stopped. What is the priority intervention by the nurse? 1 Continue to monitor the patient. 2 Check all connections for a leak in the system. 3 Investigate the chest tube for a possible occlusion. 4 Lower the drainage collector further from the chest.

3 Normal fluctuation of the water within the water-seal chamber is called tidaling. This up and down movement of water in concert with respiration reflects the intrapleural pressure changes during inspiration and expiration. Investigate any sudden cessation of tidaling, because this may signify an occluded chest tube. Monitoring the patient will occur simultaneously. The nurse could check all connections for a leak, but the most common cause is occlusion, so this would occur after checking for an occlusion. The drainage collector is likely already in a low position. Text Reference - p. 546

A 72-year-old patient is in the emergency department with a temperature of 101.4° F and a productive cough with rust-colored sputum. The nurse notifies the health care provider, understanding these findings are indicative of which condition? 1 Tuberculosis 2 Chronic heart failure (CHF) 3 Pneumonia 4 Pulmonary embolus

3 Rust-colored sputum is indicative of pneumococcal pneumonia. Tuberculosis frequently presents with a dry cough. Pink frothy sputum would be present in CHF and pulmonary edema. A pulmonary embolus would produce hemoptysis. Text Reference - p. 524

A patient with asthma who has undergone a total hip replacement complains on the third postoperative day of shortness of breath and slight chest pain and notes that "something is wrong." Temperature is 98.8° F, blood pressure 168/98 mm Hg, pulse 96, respirations 32, and oxygen saturation is 89% on room air. What is the priority nursing action? 1 Administer the prescribed antihypertensive medication and reassess in 15 minutes 2 Obtain an electrocardiogram (ECG) and administer albuterol nebulizers 3 Apply oxygen and place the patient in a semi-Fowler's position 4 Notify the health care provider and document the vital signs

3 The patient's clinical picture is consistent with pulmonary embolus, and the first action the nurse should take is to assist the patient. For this reason, the nurse should sit the patient up as tolerated and apply oxygen before notifying the health care provider. An antihypertensive or an ECG may be prescribed at a later time; however, the priority is airway, breathing, and circulation. The health care provider should be notified after the nurse has assessed the patient. Text Reference - p. 576A

What instruction does the nurse give the patient while removing the chest tube? 1 "You should sleep in a side-lying position." 2 "You should sleep with your leg elevated." 3 "You should hold your breath or bear down." 4 "You should drink juice before the procedure."

3 While removing the chest tube, the nurse instructs the patient to perform the Valsalva maneuver because it eases the process of removal. Therefore, the nurse instructs the patient to hold his or her breath or bear down. Sleeping in a side-lying position will reduce lung expansion. Therefore, the patient's condition may be aggravated. The nurse instructs the patient to elevate the leg when he or she has risk of thromboembolism. The nurse does not instruct the patient to drink juice because it may result in nausea. Text Reference - p. 547

Which statements by the student nurse indicate a need for further teaching about arterial blood gas (ABG) analysis? Select all that apply. 1 "The ABG is used to monitor oxygenation." 2 "The ABG is used to monitor acid-base balance." 3 "The ABG is used to monitor white blood cells in the blood." 4 "The ABG is used to monitor levels of carbon dioxide in the blood." 5 "The ABG is used to monitor the arterial oxygen saturation (SpO2) in the blood."

3, 5 The ABG is used to monitor oxygenation, acid-base balance, and carbon dioxide levels in the blood. The ABG is not used to monitor white blood cells or SpO2 in the blood. Text Reference - p. 478

When administering and reading the tuberculosis (TB) skin test, what measures should the nurse take? 1 Ensure that the injection is given subcutaneously. 2 Do not use a pen around the test area to mark the site. 3 Include the reddened flat areas on the skin when measuring the induration. 4 Draw a diagram of the forearm and hand and label the injection sites in the patient's chart.

4 When a skin test is administered for TB bacilli, the nurse should chart the site of administration by drawing a diagram of the forearm and hand and labeling the injection sites. The nurse should ensure that the injection is given intradermally. The nurse should circle the area with a pen and instruct the patient not to remove the mark. The diameter of the induration should be measured for reading the test. The reddened flat area is not included in the measurement. Text Reference - p. 490

Oxygen saturation obtained by pulse oximetry should be between

90% and 100%.

Additional teaching or support from others is not usually effective for

A patient for whom drug compliance is questionable

Which assessment finding in a patient admitted with acute decompensated heart failure (ADHF) requires the most immediate action by the nurse? a. Oxygen saturation of 88% b. Weight gain of 1 kg (2.2 lb) c. Heart rate of 106 beats/minute d. Urine output of 50 mL over 2 hours

ANS: A A decrease in oxygen saturation to less than 92% indicates hypoxemia. The nurse should administer supplemental oxygen immediately to the patient. An increase in apical pulse rate, 1-kg weight gain, and decreases in urine output also indicate worsening heart failure and require nursing actions, but the low oxygen saturation rate requires the most immediate nursing action.

Which action should the nurse include in the plan of care when caring for a patient admitted with acute decompensated heart failure (ADHF) who is receiving nesiritide (Natrecor)? a. Monitor blood pressure frequently. b. Encourage patient to ambulate in room. c. Titrate nesiritide slowly before stopping. d. Teach patient about home use of the drug.

ANS: A Nesiritide is a potent arterial and venous dilator, and the major adverse effect is hypotension. Because the patient is likely to have orthostatic hypotension, the patient should not be encouraged to ambulate. Nesiritide does not require titration and is used for ADHF but not in a home setting.

The nurse is caring for a patient with hypertension who is scheduled to receive a dose of metoprolol (Lopressor). The nurse should withhold the dose and consult the prescribing physician for which vital sign taken just before administration? A. O2 saturation 93% B. Pulse 48 beats/min C. Respirations 24 breaths/min D.Blood pressure 118/74 mm Hg

B Because metoprolol is a β1-adrenergic blocking agent, it can cause hypotension and bradycardia as adverse effects. The nurse should withhold the dose and consult with the health care provider for parameters regarding pulse rate limits.

Medications that block molecules involved in tumor growth

Biologic and targeted therapy

When teaching how lisinopril (Zestril) will help lower the patient's blood pressure, which mechanism of action should the nurse explain? A. Blocks β-adrenergic effects B. Relaxes arterial and venous smooth muscle C. Inhibits conversion of angiotensin I to angiotensin II D. Reduces sympathetic outflow from central nervous system

C. Lisinopril is an angiotensin-converting enzyme inhibitor that inhibits the conversion of angiotensin I to angiotensin II, which reduces angiotensin II-mediated vasoconstriction and sodium and water retention. β blockers result in vasodilation and decreased heart rate. Direct vasodilators relax arterial and venous smooth muscle. Central acting α-adrenergic antagonists reduce sympathetic outflow from the central nervous system to produce vasodilation and decreased systemic vascular resistance and blood pressure.

Considered primary treatment for small cell lung cancer (SCLC)

Chemotherapy

A 44-yr-old man is diagnosed with hypertension and receives a prescription for benazepril (Lotensin). After providing instruction, which statement by the patient indicates correct understanding? A. "If I take this medication, I will not need to follow a special diet." B. "It is normal to have some swelling in my face while taking this medication." C. "I will need to eat foods such as bananas and potatoes that are high in potassium." D. "If I develop a dry cough while taking this medication, I should notify my doctor."

D. Benazepril is an angiotensin-converting enzyme inhibitor. The medication inhibits breakdown of bradykinin, which may cause a dry, hacking cough. Other adverse effects include hyperkalemia. Swelling in the face could indicate angioedema and should be reported immediately to the prescriber. Patients taking drug therapy for hypertension should also attempt lifestyle modifications to lower blood pressure such as a reduced-sodium diet.

What is a priority nursing intervention in the care of a patient with a diagnosis of chronic venous insufficiency (CVI)? A. Application of topical antibiotics to venous ulcers B. Maintaining the patient's legs in a dependent position C. Administration of oral and/or subcutaneous anticoagulants D. Teaching the patient the correct use of compression stockings

D. Teaching the patient the correct use of compression stockings CVI requires conscientious and consistent application of compression stockings. Anticoagulants are not necessarily indicated and antibiotics, if required, are typically oral or IV, not topical. The patient should avoid prolonged positioning with the limb in a dependent position.

The patient reports tenderness when she touches her leg over a vein. The nurse assesses warmth and a palpable cord in the area. The nurse knows the patient needs treatment to prevent which sequelae? A. Pulmonary embolism B. Pulmonary hypertension C. Post-thrombotic syndrome D. Venous thromboembolism

D. Venous thromboembolism The clinical manifestations are characteristic of a superficial vein thrombosis. If untreated, the clot may extend to deeper veins, and venous thromboembolism may occur. Pulmonary embolism, pulmonary hypertension, and post-thrombotic syndrome are the sequelae of venous thromboembolism.

Pneumocystis jiroveci rarely causes pneumonia in healthy individuals but is the most common cause of pneumonia in persons with

HIV disease.

occurs due to the laceration or puncture of the lung during medical procedures.

Iatrogenic pneumothorax

Drug activated by laser light that destroys cancer cells

Photodynamic therapy

Used to prevent metastasis to the brain

Prophylactic cranial radiation

Used to treat both non-small cell lung cancer (NSCLC) and SCLC

Radiation therapy

Electric current heats and destroys tumor cells

Radiofrequency ablation

Is seen from the rupture of small blebs on the apex of the lung in patients with lung disease or smoking, as well as in tall, thin males with a family history of or a previous incident of....

Spontaneous pneumothorax

Community acquired pneumonia is most commonly caused by

Streptococcus pneumoniae

by taking a deep breath, putting the food or fluid in the mouth, swallowing while holding the breath, coughing immediately after swallowing to remove the food from the top of the vocal cord, swallowing again, then breathing you will have performed

Supraglottic swallowing

Best procedure for cure of lung cancer

Surgical therapy

occurs with mechanical ventilation and blocked chest tubes.

Tension pneumothorax

After the patient is in isolation.

The nurse will administer the antitubercular drugs

A patient with a history of chronic hypertension is being evaluated in the emergency department for a blood pressure of 200/140 mm Hg. Which patient assessment question is the priority? A. Is the patient pregnant? B. Does the patient need to urinate? C. Does the patient have a headache or confusion? Correct D. Is the patient taking antiseizure medications as prescribed?

The nurse's priority assessments include neurologic deficits, retinal damage, heart failure, pulmonary edema, and renal failure. The headache or confusion could be seen with hypertensive encephalopathy from increased cerebral capillary permeability leading to cerebral edema. In addition, headache or confusion could represent signs and symptoms of a hemorrhagic stroke. Pregnancy can lead to secondary hypertension. Needing to urinate and taking antiseizure medication do not support a hypertensive emergency.

can occur with penetrating or blunt chest trauma.

Traumatic pneumothorax

Which position should the patient assume with epistaxis?

Upright and leaning forward to prevent swallowing blood.

Continuous bubbling of pleural drainage indicates

a leak.

When assessing the patient on return to the surgical unit following a total laryngectomy and radical neck dissection, what would the nurse expect to find? a. A closed-wound drainage system b. A nasal endotracheal tube in place c. A nasogastric tube with orders for tube feedings d. A tracheostomy tube and mechanical ventilation

a. A closed-wound drainage system

Which condition contributes to secondary pulmonary arterial hypertension by causing pulmonary capillary and alveolar damage? a. COPD b. Sarcoidosis c. Pulmonary fibrosis d. Pulmonary embolism

a. COPD

Which medications would be used in four-drug treatment for the initial phase of TB? (select all that apply) a. Isoniazid (INH) d. Rifabutin (Mycobutin) b. Rifampin (Rifadin) e. Levofloxacin (Levaquin) c. Pyrazinamide (PZA) f. Ethambutol (Myambutol)

a. Isoniazid (INH) b. Rifampin (Rifadin) c. Pyrazinamide (PZA) f. Ethambutol (Myambutol)

A patient develops epistaxis upon removal of a nasogastric tube. What action should the nurse take? a. Pinch the soft part of the nose. b. Position the patient on the side. c. Have the patient hyperextend the neck. d. Apply an ice pack to the back of the neck.

a. Pinch the soft part of the nose.

The patient with chronic heart failure is being discharged from the hospital. What information should the nurse emphasize in the patient's discharge teaching to prevent progression of the disease to ADHF? a. Take medications as prescribed. b. Use oxygen when feeling short of breath. c. Only ask the physician's office questions. d. Encourage most activity in the morning when rested.

a. Take medications as prescribed. The goal for the patient with chronic HF is to avoid exacerbations and hospitalization. Taking the medications as prescribed along with nondrug therapies such as alternating activity with rest will help the patient meet this goal. If the patient needs to use oxygen at home, it will probably be used all the time or with activity to prevent respiratory acidosis. Many HF patients are monitored by a care manager or in a transitional program to assess the patient for medication effectiveness and monitor for patient deterioration and encourage the patient. This nurse manager can be asked questions or can contact the health care provider if there is evidence of worsening HF.

Cytomegalovirus (CMV) occurs in people with

an impaired immune response.

A pulmonary embolus is suspected in a patient with a deep vein thrombosis who develops hemoptysis, tachycardia, and chest pain. Diagnostic testing is scheduled. Which test should the nurse plan to teach the patient about? a. Chest x-rays b. Spiral (helical) CT scan c. Take the patient's pulse and blood pressure. d. Ventilation-perfusion lung scan

b. Spiral (helical) CT scan

When should the nurse check for leaks in the chest tube and pleural drainage system? a. There is continuous bubbling in the water-seal chamber. b. There is constant bubbling of water in the suction control chamber. c. Fluid in the water-seal chamber fluctuates with the patient's breathing. d. The water levels in the water-seal and suction control chambers are decreased.

b. There is constant bubbling of water in the suction control chamber.

You are caring for a patient with ADHF who is receiving IV dobutamine (Dobutrex). You know that this drug is ordered because it (select all that apply): a. incerases SVR b. produces diuresis c. improves contractility d. dilates renal blood vessels e. works on the B1-receptors in the heart.

c & e Rationale: Dobutamine (Dobutrex) has a positive chronotropic effect and increases heart rate and improves contractility. It is a selective β-adrenergic agonist and works primarily on the β1-adrenergic receptors in the heart. It is frequently used in the short-term management of acute decompensated heart failure (ADHF).

Priority Decision: After the health care provider sees a patient hospitalized with a stroke who developed a fever and adventitious lung sounds, the following orders are written. Which order should the nurse implement first? a. Anterior/posterior and lateral chest x-rays b. Start IV levofloxacin (Levaquin) 500 mg every 24 hr c. Sputum specimen for Gram stain and culture and sensitivity d. Complete blood count (CBC) with white blood cell (WBC) count and differential

c. Sputum specimen for Gram stain and culture and sensitivity

A patient with class 3 TB has

clinically active disease

Following a thoracotomy, the patient has a nursing diagnosis of ineffective airway clearance related to inability to cough as a result of pain and positioning. What is the best nursing intervention for this patient? a. Have the patient drink 16 oz of water before attempting to deep breathe. b. Auscultate the lungs before and after deep-breathing and coughing regimens. c. Place the patient in the Trendelenburg position for 30 minutes before the coughing exercises. d. Medicate the patient with analgesics 20 to 30 minutes before assisting to cough and deep breathe.

d. Medicate the patient with analgesics 20 to 30 minutes before assisting to cough and deep breathe.

Priority Decision: The nurse receives an evening report on a patient who underwent posterior nasal packing for epistaxis earlier in the day. What is the first patient assessment the nurse should make? a. Patient's temperature b. Level of the patient's pain c. Drainage on the nasal dressing d. Oxygen saturation by pulse oximetry

d. Oxygen saturation by pulse oximetry

Following a motor vehicle accident, the nurse assesses the driver for which distinctive sign of flail chest? a. Severe hypotension b. Chest pain over ribs c. Absence of breath sounds d. Paradoxical chest movement

d. Paradoxical chest movement

After having an MI, the nurse notes the patient has jugular venous distention, gained weight, developed peripheral edema, and has a heart rate of 108/minute. What should the nurse suspect is happening? a. ADHF b. Chronic HF c. Left-sided HF d. Right-sided HF

d. Right-sided HF An MI is a primary cause of heart failure. The jugular venous distention, weight gain, peripheral edema, and increased heart rate are manifestations of right-sided heart failure.

A male patient has chronic obstructive pulmonary disease (COPD) and is a smoker. The nurse notices respiratory distress and no breath sounds over the left chest. Which type of pneumothorax should the nurse suspect is occurring? a. Tension pneumothorax b. Iatrogenic pneumothorax c. Traumatic pneumothorax d. Spontaneous pneumothorax

d. Spontaneous pneumothorax

Which actions prevent the dislodgement of a tracheostomy tube in the first 3 days after its placement? (select all that apply) a. Provide tracheostomy care every 24 hours. b. Keep the patient in the semi-Fowler position at all times. c. Keep a same size or larger replacement tube at the bedside. d. Tracheostomy ties are not changed for 24 hours after tracheostomy procedure. e. Suction the tracheostomy tube when there is a moist cough or a decreased SpO2. f. A physician performs the first tube change, no sooner than 7 days after the tracheostomy.

d. Tracheostomy ties are not changed for 24 hours after tracheostomy procedure. e. Suction the tracheostomy tube when there is a moist cough or a decreased SpO2. f. A physician performs the first tube change, no sooner than 7 days after the tracheostomy.

What is a primary treatment goal for cor pulmonale? a. Controlling dysrhythmias b. Dilating the pulmonary arteries c. Strengthening the cardiac muscle d. Treating the underlying pulmonary condition

d. Treating the underlying pulmonary condition

those older than 65 years of age should get the Influenza vaccine how often?

each year

Rifabutin (Mycobutin) may be used as first-line treatment for patients receiving medications that

interact with rifampin.

For the initial phase of TB, the first 2 months, a four-drug regimen consists of

isoniazid (INH), rifampin (Rifadin), pyrazinamide (PZA), and ethambutol (Myambutol).

If laryngeal tumors are small, radiation is the treatment of choice because

it can be curative and can preserve voice quality.

decreasing the risk of occupational lung diseases, using masks and effective ventilation systems to reduce exposure is the

most efficient and affects the greatest number of employees.

With Flail chest the chest wall cannot provide the support for ventilation and the injured segment will

move paradoxically to the stable portion of the chest (in on expiration; out on inspiration).

Airborne infection isolation is required for active disease until the patient is noninfectious, indicated by

negative sputum smears.

Absent breath sounds occur following

pneumothorax or hemothorax.

A patient began taking antitubercular drugs a week ago. The nurse reviews the patient's medical record and learns that the patient has a 10-year history of consuming one standard drink of alcohol three times a week. The patient states, "In the last week, my urine turned orange and I am very worried about it." How should the nurse respond? 1 Inform the patient that it is one of the side effects of the antitubercular drug rifampin. 2 Recognize that the tuberculosis may have spread to the liver, and further medical consultation is required. 3 Recognize that the liver may be damaged due to alcohol, and so a liver function test should be performed. 4 Instruct the patient to stop taking antitubercular drugs immediately and consult the primary health care provider

.1 A nurse should be aware of some of the common side effects of antitubercular drugs like rifampin, one of which is orange discoloration of body fluids such as urine, sweat, tears, and sputum. It may also cause hepatitis. Liver damage can lead to jaundice, which usually presents as yellowish discoloration of urine and sclera. However it is highly unlikely that tuberculosis has spread to the liver. The alcohol intake of the patient is within normal limits, and so it is not correct to say that alcohol may have damaged the liver. It is also inappropriate to advise the patient to stop taking antitubercular drugs. Text Reference - p. 531

The nurse cares for a patient with a diagnosis of tuberculosis. Which assessment finding best indicates that the patient has been following the prescribed treatment plan? 1 Negative sputum cultures 2 Clear breath sounds bilaterally 3 Decrease in the number of coughing episodes 4 Conversion of the Mantoux test from positive to negative

1 A patient's sputum is expected to convert to negative within three months of the beginning of treatment. If it does not, the patient is either not taking the medication or has drug-resistant organisms. Bilaterally clear breath sounds and a decrease in coughing are good indications that the patient is following the prescribed plan, but they are not as confirmatory as negative sputum cultures. Once a person has been exposed to the tuberculosis-causing organism, the Mantoux test will always elicit a positive result. Text Reference - p. 533

Which complication does the nurse expect when a patient with fractured ribs is wearing a binder on the chest? 1 Atelectasis 2 Chylothorax 3 Hemopneumothorax 4 Pulmonary embolism

1 Applying a binder on fractured ribs can reduce the chest expansion and retain secretions. Therefore, the patient will have the risk of atelectasis. Chylothorax is complication of chest trauma, which results from disruption of the thoracic duct. Hemopneumothorax is the presence of blood in the pleural cavity due to chest trauma. Pulmonary embolism is a blockage of pulmonary a blood vessel due to thrombus formation; this complication is not associated with application of a binder.

A nurse is caring for a patient who experiences shortness of breath, severe productive cough, and fever. The sputum, which is yellow, has been sent for laboratory testing, but the lab report is pending. What should be the next step in managing the patient? 1 Administer antibiotics. 2 Wait for the lab report. 3 Retake sputum sample and send to another lab. 4 Refer the patient for chest physiotherapy.

1 Culture and Gram stain of sputum is required for prescribing specific antibiotics. However, if there is a delay in obtaining the lab reports, antibiotic administration should not be delayed. Deferring the antibiotics may lead to increased morbidity and mortality because the infection can worsen. Taking the sample again to send to a different lab would not be helpful because the lab will take a similar amount of time to provide the report. Chest physiotherapy can be advised later once the antibiotic therapy is started. Text Reference - p. 525

The patient with Parkinson's disease has a pulse oximetry reading of 72% but the patient is not displaying any other signs of decreased oxygenation. What most likely is contributing to the patient's low SpO2 level? 1 Motion 2 Anemia 3 Dark skin color 4 Thick acrylic nails

1 Motion is the most likely cause of the low SpO2 for this patient with Parkinson's disease. Anemia, dark skin color, and thick acrylic nails, as well as low perfusion, bright fluorescent lights, and intravascular dyes also may cause an inaccurate pulse oximetry result. There is no mention of these or any reason to suspect these in this question. Text Reference - p. 478

A patient with recurrent shortness of breath has just had a bronchoscopy. What is a priority nursing action immediately following the procedure? 1 Monitor the patient for laryngeal edema. 2 Assess the patient's level of consciousness (LOC). 3 Monitor and manage the patient's level of pain. 4 Assess the patient's heart rate and blood pressure

1 Priorities for assessment are the patient's airway and breathing, both of which may be compromised after bronchoscopy by laryngeal edema. These assessment parameters supersede the importance of LOC, pain, heart rate, and blood pressure, although the nurse should be assessing these also. Text Reference - p. 492

A patient is taking sildenafil for treatment of pulmonary hypertension. The nurse who is monitoring the patient finds that the patient's blood pressure has dropped to 80/60 mm Hg. The patient's current home medications include multivitamins, thyroxine, and nitroglycerin for comorbid conditions. The nurse contacts the primary health care provider and expects what change in the patient's treatment plan? 1 Stopping sildenafil 2 Stopping thyroxine 3 Lung transplantation 4 Administering intravenous fluids

1 Sildenafil should not be given to patients who are on nitroglycerin, because it can lead to refractory hypotension. Therefore sildenafil should be stopped immediately. Thyroxine does not interact with sildenafil and also does not cause hypotension. Lung transplantation is reserved for patients with pulmonary hypertension who do not respond to drug therapy. Administering intravenous fluids without correcting the cause will not help the patient. Text Reference - p. 555

Which sized chest tube does the nurse use to drain air from the patient's chest? 1. 14F 2. 26F 3. 36F 4. 40F

1 Small tubes having the size in the range of 12F to 24F are used to drain air from the patient's chest. Medium tubes in the range of 24F to 36F are used for draining fluid from the patient's chest. Large tubes ranging from 36F to 40F are used for draining blood from the patient's chest.

Which defense mechanism is effective for removing secretions in the main airways? 1 Cough reflex 2 Mucociliary escalator 3 Alveolar macrophages 4 Reflex bronchoconstriction

1 The cough reflex is a protective reflex action that clears the airway by a high pressure, high velocity flow of air. Coughing is effective in removing secretions in the main airways. Below the larynx, movement of mucus is accomplished by the mucociliary clearance system or the mucociliary escalator. Secretions below the subsegmental level must be moved upward by the mucociliary escalator before they can be removed by coughing. Alveolar macrophages rapidly phagocytize foreign particles, such as bacteria. The debris is moved by the cilia in the bronchi for removal. Reflex bronchoconstriction is a protective mechanism, triggered by inhaling irritating substances. Text Reference - p. 481 v

A nurse is caring for a patient with a pulmonary embolism who is on warfarin therapy. Which parameter should the nurse monitor in this patient? 1 Hematomas 2 Polycythemia 3 B type natriuretic peptide 4 White blood cell differentials

1 The nurse should monitor a patient with pulmonary embolism who is on warfarin therapy for complications such as hematomas, bruising, and bleeding. Polycythemia in a patient with cor pulmonale is secondary to chronic obstructive pulmonary disease. B type natriuretic peptide and serum troponin levels are elevated in patients with increased mortality due to pulmonary embolism. White blood cell differentials are helpful to assess pulmonary embolism. Text Reference - p. 552

Which physical assessment finding in a patient with a lower respiratory problem best supports the nursing diagnosis of ineffective airway clearance? 1 Basilar crackles 2 Respiratory rate of 28 3 Oxygen saturation of 85% 4 Presence of greenish sputum

1 The presence of adventitious breath sounds indicates that there is accumulation of secretions in the lower airways. This would be consistent with a nursing diagnosis of ineffective airway clearance because the patient is retaining secretions. The rapid respiratory rate, low oxygen saturation, and presence of greenish sputum may occur with a lower respiratory problem, but do not support definitely the nursing diagnosis of ineffective airway clearance. Text Reference - p. 524

The patient is calling the clinic complaining of a cough. What assessment should be made first before the nurse advises the patient? 1 Cough sound, sputum production, and pattern 2 Frequency, a family history, and hematemesis 3 Smoking, medications, and residence location 4 Weight loss, activity tolerance, and orthopnea

1 The sound of the cough, sputum production, and description, as well as pattern, of the cough's occurrence (including acute or chronic) and what its occurrence is related to are the first assessments to be made to determine the severity. Frequency of the cough will not provide a lot of information. Family history can help to determine a genetic cause of the cough. Hematemesis is vomiting blood and is not as important as hemoptysis. Smoking is an important risk factor for chronic obstructive lung disease (COPD) and lung cancer and may cause a cough. Medications may or may not contribute to a cough as well as residence location. Weight loss, activity intolerance, and orthopnea may be related to respiratory or cardiac problems, but are not as important when dealing with a cough. Text Reference - p. 482

Which statement is true regarding a water seal chamber? 1 It contains 2 cm of water. 2 It acts as a two-way valve. 3 It receives fluid and air from the pleural space. 4 It applies suction to the chest drainage system

1 The water seal chamber is the second chamber of the chest drainage system. It contains 2 cm of water, which acts as a one-way valve. The first chamber of the drainage system receives fluid and air from the pleural space. The third chamber applies suction to the chest drainage system.

During a respiratory assessment, the nurse notes coarse crackles upon auscultation of the lung fields. Which diagnoses present with this assessment finding? Select all that apply. 1 Pneumonia 2 Heart failure 3 Cystic fibrosis 4 Bronchospasm 5 Interstitial edema

1, 2 Coarse crackles are often auscultated in patients diagnosed with pneumonia or heart failure. Rhonchi are auscultated in patients diagnosed with cystic fibrosis. Wheezes are auscultated when the patient is experiencing bronchospasm. Discontinuous, low-pitched lung sounds are auscultated in patients experiencing interstitial edema. Text Reference - p. 489

The nurse is caring for a patient who has a nasogastric tube. What actions should the nurse perform to prevent aspiration in this patient? Select all that apply. 1 Elevate the head of the bed 30 to 45 degrees. 2 Monitor gastric residual volumes. 3 Encourage the patient to sit upright for all meals. 4 Lower the head of the bed to 10 degrees. 5 Feed the patient in a reclined position.

1, 2, 3 Although the nasogastric tubes are small, there is an increased risk for aspiration pneumonia; to prevent it, the head of the bed should be elevated to 30 to 45 degrees, gastric residual volumes should be monitored, and the patient should be made to sit upright for all meals. Lowering the head of the bed and reclining while eating are not advisable, because these positions can increase the risk of aspiration. Text Reference - p. 527

On auscultation of a patient's lungs, which breath sounds would the nurse consider normal? Select all that apply. 1 Loud, high-pitched sounds resembling air blowing through a hollow pipe 2 Soft, low-pitched, gentle, rustling sounds heard over all lung areas except the major bronchi 3 Medium-pitched sounds heard anteriorly over the mainstem bronchi on either side of the sternum 4 The patient repeats the phrase "ninety-nine," and the words are easily understood and are clear and loud through the chest. 5 The patient whispers "one-two-three," and the almost inaudible voice is transmitted clearly and distinctly.

1, 2, 3 Bronchial, vesicular, and bronchovesicular sounds are normal breath sounds. Bronchial sounds are loud and high-pitched and resemble air blowing through a hollow pipe. Vesicular sounds are soft, low-pitched, gentle, rustling sounds heard over all lung areas except the major bronchi. Bronchovesicular sounds are medium-pitched sounds heard anteriorly over the mainstem bronchi on either side of the sternum. Bronchophony is an abnormal breath sound and is considered positive (abnormal) if the patient repeats the phrase "ninety nine" and the words are easily understood and are clear and loud. Whispered pectoriloquy is also an abnormal breath sound and is considered positive (abnormal) when the patient whispers "one-two-three," and the almost inaudible voice is transmitted clearly and distinctly. Text Reference - p. 487

The nurse is caring for a patient who is a smoker and is diagnosed with chronic obstructive pulmonary disease (COPD). Which sputum characteristics, if present in the patient, may need further evaluation? Select all that apply. 1 Frothy 2 Foul odor 3 Pink tinged 4 Brown specks

1, 2, 3 COPD may result in whitish to yellowish sputum, however, any change in the baseline characteristics of the sputum should be reported. Frothy sputum may indicate pulmonary edema and needs further evaluation. A foul odor in the sputum indicates presence of infection and needs immediate medical intervention. Pink-tinged sputum may indicate pulmonary edema and the patient may need further evaluation. Sputum with brown specks is a common finding in a person who smokes. Yellowish sputum is a normal finding in COPD. Text Reference - p. 482

The nurse makes a nursing diagnosis of "impaired gas exchange" for a patient with pneumonia based upon which physical-assessment findings? Select all that apply. 1 SpO2 of 85% 2 PaO2 of 65 mm Hg 3 Absent breath sounds in right lung lobes 4 Presence of thick yellow mucus 5 Respiratory rate 24 breaths/minute

1, 2, 3 Impaired gas exchange is evidenced by low oxygen saturation and elevated PaCO2 with absent breath sounds. Yellow mucus would indicate clearance of secretions. An increased respiratory rate does not imply impaired gas exchange. Text Reference - p. 525

The nurse is caring for the patient with a pulmonary embolism. Which factor(s) are associated with a pulmonary embolism (PE)? Select all that apply. 1 Pregnancy 2 Pelvic surgery 3 Immobility 4 Herbal therapy

1, 2, 3 Pregnancy, pelvic surgery, and immobility are major risk factors associated with a pulmonary embolism. Risk factors among many for PE include immobility, pelvic surgery, pregnancy, oral contraceptives and hormone therapy. Herbal therapy is incorrect because herbal therapy is not associated with the development of a pulmonary embolism. Text Reference - p. 578

When caring for a patient with tuberculosis, what measures should the nurse instruct the patient to take to avoid the spread of infection? Select all that apply. 1 Cover the nose and mouth with a tissue while coughing and sneezing. 2 Throw used tissues in a paper bag and dispose with the trash. 3 Carefully wash hands after handling sputum and soiled tissues. 4 Wear a standard isolation mask when outside the patient's room. 5 Get out of bed and move freely about the hospital to keep up strength. 6 Drink plenty of water and maintain an erect posture

1, 2, 3, 4 In order to prevent the spread of infection, patients with tuberculosis should be encouraged to cover the nose and mouth with tissues while coughing and sneezing, to throw used tissues in a paper bag and dispose of them with the trash, to carefully wash hands after handling sputum and soiled tissues, and to wear a standard isolation mask while moving out of their room. Increasing the frequency of prolonged visits to other parts of the hospital is not advisable, because it can increase the chances of infection spread; instead, such visits should be limited. Drinking plenty of water and maintaining erect posture have no effect on controlling infection. Text Reference - p. 533

The nurse is caring for a patient with a pleural effusion. What nursing interventions are appropriate when preparing this patient for a thoracentesis? Select all that apply. 1 Ensure that the informed consent was signed. 2 Position patient upright with elbows on an over bed table with feet supported. 3 Instruct the patient to cough vigorously during the procedure. 4 Instruct the patient not to eat anything for four hours before the procedure. 5 Obtain chest x-ray after the procedure.

1, 2, 4 For a thoracentesis, the nurse should ensure that the patient's informed consent was signed. The patient should be positioned upright with elbows on an over-bed table and feet supported. This position gives appropriate access for needle insertion. A chest x-ray is obtained after the procedure to rule out a pneumothorax. The patient should be instructed not to talk or cough during the procedure, because it can cause injury by displacement of the needle. NPO status, or withholding food and drink, is not required for thoracentesis.

Which nursing interventions will the nurse implement to increase mucus production in a 72-year-old patient with bronchitis? Select all that apply. 1 Humidify the oxygen 2 Increase fluid intake to 3 L per day if tolerated 3 Administer a cough suppressant 4 Elevate the head of the bed to 45 degrees 5 Teach pursed lip breathing

1, 2, 4 Several interventions may help the patient expectorate mucus, including increasing fluid intake, which will liquefy the secretions so that the patient can expectorate them more easily, humidifying the oxygen to loosen secretions, and elevating the head of the bed to increase movement of mucous. Pursed lip breathing and a cough suppressant will not promote mucus production and expectoration. Text Reference - p. 527

A nurse is assessing a patient with dengue. The patient suddenly develops acute respiratory distress syndrome (ARDS). Which patient findings indicate inadequate oxygenation? Select all that apply. 1 Cyanosis 2 Diaphoresis 3 Anemia 4 Tachypnea 5 Hypertension

1, 2, 4 The symptoms of inadequate oxygenation in the patient include cyanosis, diaphoresis, and tachypnea. Cyanosis indicates inadequate perfusion due to compromised oxygenation. Diaphoresis and tachypnea occur due to sympathetic stimulation to compensate for inadequate oxygenation. Anemia occurs gradually and does not suddenly cause inadequate oxygenation. Hypertension does not indicate inadequate oxygenation in the patient. Text Reference - p. 479

To promote airway clearance in a patient with pneumonia, what should the nurse instruct the patient to do? Select all that apply. 1 Maintain adequate fluid intake 2 Splint the chest when coughing 3 Maintain a 30-degree elevation 4 Maintain a semi-Fowler's position 5 Instruct patient to cough at end of exhalation

1, 2, 5 Maintaining adequate fluid intake liquefies secretions, allowing easier expectoration. The nurse should instruct the patient to splint the chest while coughing. This will reduce discomfort and allow for a more effective cough. Coughing at the end of exhalation promotes a more effective cough. The patient should be positioned in an upright sitting position (high-Fowler's) with head slightly flexed. Text Reference - p. 527

A patient is scheduled for a biopsy through bronchoscopy. What nursing interventions are appropriate for this patient? Select all that apply. 1 Ensure that an informed consent has been signed. 2 Assess blood urea nitrogen (BUN) and serum creatinine levels. 3 Administer sedative, if prescribed. 4 Instruct patient to remove all metal from the body. 5 Instruct patient to be on NPO status for 6 to 12 hours before the test.

1, 3, 5 The nurse should ensure the consent form has been signed and administer sedatives if prescribed. For a bronchoscopy, the patient is required to be on NPO for 6 to 12 hours before the procedure. Renal function is not related to bronchoscopy, so assessment of BUN and serum creatinine levels is not needed. Removing metals from the body may not be necessary, because the procedure does not involve x-rays. Text Reference - p. 492

When assessing a patient's sleep-rest pattern related to respiratory health, about what should the nurse ask the patient? Select all that apply. 1 Do you have trouble falling asleep? 2 Do you need to urinate during the night? 3 Do you awaken abruptly during the night? 4 Do you sleep more than eight hours per night? 5 Do you need to sleep with the head elevated?

1, 3, 5 The patient with sleep apnea may have insomnia or abrupt awakenings. Patients with cardiovascular disease (e.g., heart failure that may affect respiratory health) may need to sleep with the head elevated on several pillows (orthopnea). Sleeping more than eight hours per night or needing to urinate during the night is not indicative of impaired respiratory health. Text Reference - p. 483

The nurse is assessing a patient who was admitted from a nursing home with suspected tuberculosis (TB). Which of these are initial manifestations of tuberculosis? Select all that apply. 1 Anorexia 2 Dyspnea 3 Night sweats 4 Hemoptysis 5 Low-grade fever 6 Unexplained weight loss

1, 3, 5, 6 Active TB disease may manifest initially with constitutional symptoms such as fatigue, malaise, anorexia, unexplained weight loss, low-grade fevers, and night sweats. Dyspnea is a late symptom that may signify considerable pulmonary disease or a pleural effusion. Hemoptysis, which occurs in less than 10% of patients with TB, is also a late symptom. Text Reference - p. 529

What are age-related changes to the respiratory system's defense mechanisms? Select all that apply. 1 Decreased cilia function 2 Decreased chest wall compliance 3 Decreased response to hypoxemia 4 Decreased cell-mediated immunity 5 Decreased respiratory muscle strength

1, 4 There are three categories of age-related changes that impact the respiratory system, including changes in structure, defense mechanisms, and respiratory control. A decrease in cilia function and cell-mediated immunity are both age-related defense mechanism changes. Decreased chest wall compliance is a structural change. Decreased response to hypoxemia is a respiratory control change. Decreased respiratory muscle strength is an age-related structural change. Text Reference - p. 481

A patient had an intradermal tuberculin skin test (Mantoux) administered 48 hours ago. The nurse assesses the injection site and identifies a 12-mm area of palpable induration. How should the nurse interpret this result? 1 Definitive evidence that the patient does not have tuberculosis 2 A significant indication that the patient has been exposed to tuberculosis 3 Delayed hypersensitivity with a high likelihood of infection with tuberculosis 4 A negative test that cannot be interpreted as ruling out the presence of tuberculosis

2 An area of 12 mm of induration at the injection site 48 hours after a Mantoux test is considered significant for a past or current tuberculin infection. An induration of less than 5 mm is considered a negative result. The other answer options are incorrect conclusions related to the findings. Text Reference - p. 530

A patient is brought to the emergency department with chest tightness and acute dyspnea after an afternoon of gardening. As the nurse auscultates the patient's lungs, which finding would indicate a need for asthma testing? 1 Rhonchi 2 Wheezes 3 Fine crackles 4 Coarse crackles

2 Asthma involves bronchospasms, which can be triggered by many factors including pollens inhaled during outdoor activities such as gardening. Wheezes are continuous, high-pitched squeaking or musical sounds caused by the rapid vibration and narrowing of bronchial walls. If the patient has wheezing sounds during auscultation, it indicates the patient may have asthma. Rhonchi sounds are continuous rumbling, snoring, or rattling sounds caused by obstruction of large airways with secretions. This would be seen in instances of cystic fibrosis. Fine crackles are series of short-duration, discontinuous, high-pitched sounds heard just before the end of inspiration, as seen in cases of pulmonary fibrosis and interstitial edema. Coarse crackles are long-duration, discontinuous, and low-pitched and they are usually caused by air passing through an airway intermittently occluded by mucus, unstable bronchial walls, or folds of mucosa. Coarse crackles can be heard in conditions such as heart failure and pulmonary edema. Text Reference - p. 489

Which strategy by the nurse would be most helpful in treating a patient with pneumonia and asthma who is experiencing chills? 1 Apply heat to the patient's posterior neck 2 Provide a blanket to the patient 3 Encourage the patient to bathe in hot water 4 Administer acetaminophen (Tylenol) 650 mg as prescribed

2 Chills often occur in cycles and last for 10 to 30 minutes at a time. They usually signal the onset of an increase in temperature. For this reason, the nurse should provide a light blanket for comfort, but avoid overheating the patient with heat on the neck or bathing in hot water. Acetaminophen would be used if the patient became febrile following the chills. Text Reference - p. 528

The nurse is caring for a patient with inflammation associated with sharp and abrupt pain upon inspiration. Which action of the nurse would be beneficial to the patient? 1 Restricting sodium in the patient's diet 2 Teaching the patient to splint the ribs while coughing 3 Encouraging the patient to take deep and long breaths 4 Encouraging the patient to exercise regularly for 30 minutes

2 Inflammation associated with sharp and abrupt pain upon inspiration indicates pleurisy. This occurs due to friction rub of the inflamed pleura during inspiration. Analgesics are helpful to relieve pain, but the nurse has to teach a patient with pleurisy to lay on the affected side or to splint the rib cage to provide comfort. A sodium-rich diet is indicated in the patient with cor pulmonale and pulmonary hypertension to reduce the incidence of heart failure. Deep inspirations or breathing exercises for 30 minutes regularly may trigger the patient's condition and should be avoided.

The nurse is caring for a patient with a diagnosis of lung abscess. The nurse knows that: 1 Hemoptysis rarely occurs. 2 Purulent sputum often is dark brown 3 Physical examination reveals hyperresonance and fremitus 4 Clinical manifestations related to anaerobic organisms usually occur more acutely over a period of a few days

2 Purulent sputum is dark brown, foul smelling, and foul tasting. Hemoptysis is common. Anaerobic organisms are the primary cause of lung abscesses and clinical manifestations occur slowly over a period of weeks to months. Physical examination shows dullness and decreased breath sounds. Test-Taking Tip: Do not panic while taking an exam! Panic will only increase your anxiety. Stop for a moment, close your eyes, take a few deep breaths, and resume review of the question. Text Reference - p. 534

When planning appropriate nursing interventions for a patient with metastatic lung cancer and a 60-pack-per-year history of cigarette smoking, the nurse recognizes that the smoking most likely has decreased the patient's underlying respiratory defenses because of impairment of: 1 Cough reflex 2 Mucociliary clearance 3 Reflex bronchoconstriction 4 Ability to filter particles from the air

2 Smoking decreases the mucociliary action in the tracheobronchial tree, resulting in impaired clearance of respiratory secretions and particles, chronic cough, and frequent respiratory infections. Cough reflex, reflex bronchoconstriction, and ability to filter particles from the air do not increase underlying respiratory defenses. Test-Taking Tip: Once you have decided on an answer, look at the stem again. Does your choice answer the question that was asked? If the question stem asks "why," be sure the response you have chosen is a reason. If the question stem is singular, then be sure the option is singular, and the same for plural stems and plural responses. Many times, checking to make sure that the choice makes sense in relation to the stem will reveal the correct answer. Text Reference - p. 536

The health care provider has prescribed intravenous (IV) vancomycin for a patient with pneumonia. Which action should the nurse perform first? 1 Obtain a full set of vital signs 2 Obtain sputum cultures for sensitivity 3 Draw a blood specimen to evaluate the white blood cell count 4 Administer the antibiotic over at least 60 minutes

2 The nurse should ensure that the sputum for culture and sensitivity has been sent to the laboratory before administering the antibiotic. It is important that the organisms be correctly identified (in the culture) before their numbers are affected by the antibiotic; the test also will determine whether the proper antibiotic has been prescribed (sensitivity testing). Vital signs and white blood cell count measurement can be assessed following the obtainment of sputum cultures. Timing of antibiotic administration should be based upon the institution's policy. Text Reference - p. 525

A nurse is caring for a patient experiencing respiratory distress. The patient has a respiratory rate of 24 breaths/minute. Arterial blood gas (ABG) analysis reveals a pH of 6, and the spirometric measurement of tidal volume is 600 mL. What do these values and symptoms indicate? 1 Metabolic acidosis 2 Respiratory acidosis 3 Respiratory alkalosis 4 Metabolic alkalosis

2 The patient has an increased respiratory rate with increased tidal volume; pH is 6, which indicates respiratory acidosis. In metabolic acidosis, the symptoms include rapid, regular, and deep inspirations. In respiratory and metabolic alkalosis, pH exceeds 7.5. Text Reference - p. 480

The nurse is caring for a patient with shortness of breath and a respiratory rate of 28 breaths/minute. Which is the most preferred method to auscultate the chest of a patient in this condition? 1 Starting at the apices 2 Starting at the lung bases 3 Listening to and comparing opposite areas of the chest 4 Listening to a complete inspiratory and expiratory cycle

2 The patient is in respiratory distress. Starting at the lung bases is the most preferred method in a patient in respiratory distress, because the increased respiratory rate and shortness of breath may tire the patient easily. Generally, auscultation should proceed from the lung apices to the bases, comparing opposite areas of the chest, unless the patient is in respiratory distress or will tire easily. During any auscultation, the nurse listens to at least one cycle of inspiration and expiration with each placement of the stethoscope. Text Reference - p. 490

How will splinting the incision with a pillow benefit a patient who underwent surgery to repair chest trauma? 1 It will reduce pain perception. 2 It will facilitate deep breathing. 3 It will reduce the risk of air leak. 4 It will increase perfusion at the site

2 The patient will have difficulty breathing after surgery due to the incision on the chest. Splinting the incision facilitates deep breathing. The nurse administers analgesics to reduce pain. An occlusive dressing is applied over the site of surgery to reduce air leakage. The nurse instructs the patient to perform range of motion exercise to increase perfusion or oxygen supply to the injured site. Text Reference - p. 548

What does the nurse interpret from finding excessive bubbling in the water seal chamber while monitoring a patient who is on chest tube drainage? 1 The patient may have lung expansion. 2 The patient may have a bronchopleural leak. 3 The patient may have a large pleural air leak. 4 The patient may have an infection at the drainage site.

2 The patient with chest trauma is in need of chest tube drainage. Excess bubbling in the water seal chamber may cause air leakage from the water seal chamber and may result in bronchopleural leak. Lung expansion in the water seal chamber is indicated by tidaling. An absence of bubbles in the chamber may indicate a large pleural air leak. The presence of inflammation, erythema at the site, and an increase in white blood cell count indicates infection.

The nurse is assessing a patient diagnosed with tuberculosis. Which assessment finding supports this diagnosis? 1 Wheezing 2 Hemoptysis 3 Gray sputum 4 Slightly whitish sputum

2 Tuberculosis is characterized by hemoptysis, which is the act of coughing up blood or blood-tinged sputum from the respiratory tract. Wheezing is the term used to describe the musical sounds auscultated during assessment; it indicates some degree of airway obstruction that occurs with asthma and emphysema. Grey sputum often occurs in patients who smoke cigarettes. Clear sputum, slightly whitish sputum, and viscous sputum are often normal findings. Text Reference - p. 483

Which questions are appropriate when assessing the effects of the patient's respiratory diagnosis on activity-exercise patterns? Select all that apply. 1 "Are you ever incontinent of urine when you cough?" 2 "Do you have trouble walking due to shortness of breath?" 3 "Does your spouse wake you in the middle of the night due to snoring?" 4 "How many flights of stairs can you walk up before you are short of breath?" 5 "Do you ever feel full very quickly when eating due to your breathing issues?"

2, 4 When assessing the effects that a respiratory diagnosis has on activity-exercise patterns, the nurse will ask the patient if walking is impacted by dyspnea and how many flights of steps the patient can walk up before dyspnea occurs. Asking the patient about urinary incontinence with coughing is appropriate when assessing elimination patterns. Asking the patient if the spouse wakes him or her up in the middle of the night due to snoring will assess sleep-rest patterns. Asking the patient if he or she feels full quickly when eating assesses the patient's nutritional-metabolic pattern.

A patient with pneumonia is being treated at home and has reported fatigue to the nurse. What instructions should the nurse include when teaching the patient about care and recovery at home? Select all that apply. 1 Resume work to build strength. 2 Get adequate rest. 3 Restrict fluid intake. 4 Avoid alcohol and smoking. 5 Take every dose of the prescribed antibiotic

2, 4, 5 To ensure complete recovery after pneumonia, the patient should be advised to rest, avoid alcohol and smoking, and take every dose of the prescribed antibiotic. The patient should not resume work if feeling fatigued and should be encouraged to drink plenty of fluids during the recovery period. Test-Taking Tip: The presence of absolute words and phrases can also help you determine the correct answer to a multiple-choice item. If answer choices contain an absolute (e.g., none, never, must, cannot), be very cautious. Remember that there are not many things in the world that are absolute, and in an area as complex as nursing, an absolute may be a reason to eliminate it from consideration as the best choice. This is only a guideline and should not be taken to be true 100% of the time; however, it can help you reduce the number of choices. Text Reference - p. 528

While assessing a patient, the nurse finds that the partial pressure of oxygen (PaO2) is 39 percent and the peripheral capillary oxygen saturation (SpO2) is 72 percent. Which complications are associated with these assessment findings? Select all that apply. 1 Atelectasis 2 Tissue hypoxia 3 Pleural effusion 4 Pulmonary edema 5 Cardiac arrhythmia

2, 5 The patient's PaO2 and SpO2 indicate inadequate oxygenation. Complications associated with these findings include tissue hypoxia and cardiac arrhythmia. Atelectasis, pleural effusion, and pulmonary edema are not complications associated with these data. Text Reference - p. 480

Which patient would benefit from a lung transplant? 1 A patient with poor nutritional status 2 A patient with chronic active hepatitis B 3 A patient with alpha1-antitrypsin deficiency 4 A patient with human immunovirus infection

3 A patient with alpha1-antitrypsin deficiency has an inability to restore the lung elasticity; therefore, lung transplantation is indicated in this patient. Lung transplantation involves immunosuppressant treatment, which may not be tolerated in patients with poor nutritional status, chronic active hepatitis B, and human immunovirus infection. Text Reference - p. 556

Which clinical manifestation should the nurse expect to find during assessment of a patient admitted with pneumonia? 1 Hyperresonance on percussion 2 Vesicular breath sounds in all lobes 3 Increased vocal fremitus on palpation 4 Fine crackles in all lobes on auscultation

3 A typical physical examination finding for a patient with pneumonia is increased vocal fremitus on palpation. Other signs of pulmonary consolidation include bronchial breath sounds, egophony, and crackles in the affected area. With pleural effusion, there may be dullness to percussion over the affected area.

Which is the term used to describe abnormal breath sounds? 1 Vesicular 2 Bronchial 3 Adventitious 4 Bronchovesicular

3 Adventitious is the term used to describe abnormal breath sounds such as crackles, rhonchi, wheezes, and a pleural friction rub. The three normal breath sounds are vesicular, bronchovesicular, and bronchial sounds. Vesicular sounds are relatively soft, low-pitched, gentle, rustling sounds. Bronchial sounds are louder and higher pitched; they resemble air blowing through a hollow pipe. Bronchovesicular sounds have a medium pitch and intensity and are heard anteriorly over the main stem bronchi on either side of the sternum and posteriorly between the scapulae. Text Reference - p. 488

The nurse is providing care to a patient who is diagnosed with pneumonia. The patient smokes one pack of cigarettes per day. Which respiratory defense mechanism may have failed, causing the patient's diagnosis? 1 Cough reflex 2 Filtration of air 3 Alveolar macrophages 4 Mucociliary clearance system

3 Alveolar macrophages rapidly phagocytize inhaled foreign particles, such as bacteria, and often fail as a result of cigarette smoking. The cough reflex, filtration of air, and mucociliary clearance are not the respiratory defense mechanisms that may have failed and caused pneumonia. Text Reference - p. 480

The patient is hospitalized with pneumonia. Which diagnostic test should be used to measure the efficiency of gas transfer in the lung and tissue oxygenation? 1 Thoracentesis 2 Bronchoscopy 3 Arterial blood gases 4 Pulmonary function tests

3 Arterial blood gases are used to assess the efficiency of gas transfer in the lung and tissue oxygenation, as is pulse oximetry. Thoracentesis is used to obtain specimens for diagnostic evaluation, remove pleural fluid, or instill medication into the pleural space. Bronchoscopy is used for diagnostic purposes, to obtain biopsy specimens, and to assess changes resulting from treatment. Pulmonary function tests measure lung volumes and airflow to diagnose pulmonary disease, monitor disease progression, evaluate disability, and evaluate response to bronchodilators. Text Reference - p. 491

A nurse preparing educational information about lung cancer notes that the primary risk factor related to the development of lung cancer is: 1 Genetics 2 Chewing tobacco 3 Cigarette smoking 4 Occupational exposure

3 As many as 90% of patients with lung cancer have a history of cigarette smoking. Cigarette smoke contains several organ-specific carcinogens. Genetics and occupational exposure are risks but not as high as cigarette smoking. Chewing tobacco is not associated with lung cancer; it is related to oral cancer. Text Reference - p. 535

A new nurse is observed caring for a patient who has a chest tube in place after a partial lobectomy. Which action by the nurse is inappropriate and requires a more experienced nurse to intervene? 1 Positioning the patient in a semi-Fowler's position 2 Encouraging the patient to increase his fluid intake 3 Clamping the chest tube while the patient is ambulating to the bathroom 4 Administering the patient's prescribed narcotic analgesic before activity

3 Clamping of chest tubes is generally contraindicated. Clamping would allow a buildup of secretions or air in the pleural space and inhibit lung expansion. Patients with chest tubes are usually placed in the semi-Fowler's position for the sake of comfort and ease of breathing. Fluid intake is not a major concern in light of the information provided. Administering the patient's prescribed narcotic analgesic before activity is an expected standard of care for this patient. Text Reference - p. 546

A patient presents in the emergency department with dyspnea. The nurse notes a bluish discoloration of the patient's lips, fine crackles on auscultation, and dullness upon percussion of the lung fields. Which diagnosis does the nurse anticipate? 1 Asthma 2 Pleural effusion 3 Pulmonary edema 4 Pulmonary fibrosis

3 Dyspnea, cyanosis, fine crackles, and dullness on percussion all support the diagnosis of pulmonary edema. Wheezing and hyperresonance on percussion support the diagnosis of asthma. Tachypnea, diminished or absent breath sounds, and dullness on percussion support the diagnosis of pleural effusion. Tachypnea, crackles, and resonance on percussion support the diagnosis of pulmonary fibrosis.

When the patient is diagnosed with a lung abscess, what should the nurse teach the patient? 1 Lobectomy surgery usually is needed to drain the abscess. 2 Intravenous (IV) antibiotic therapy will be used for a prolonged period of time. 3 Oral antibiotics will be used when the patient and x-ray show evidence of improvement. 4 No further culture and sensitivity tests are needed if the patient takes the medication as prescribed.

3 IV antibiotics are used until the patient and x-ray show evidence of improvement. Then oral antibiotics are used for a prolonged period of time. Lobectomy surgery is needed only when reinfection of a large cavitary lesion occurs or to establish a diagnosis when there is evidence of a neoplasm or other underlying problem. Culture and sensitivity testing is done during the course of antibiotic therapy to ensure that the infecting organism is not becoming resistant to the antibiotic as well as at the completion of the antibiotic therapy. Text Reference - p. 534

The nurse is providing care to a patient who is being mechanically ventilated due to diaphragm paralysis. Which is the most likely reason for the current clinical manifestations and treatment plan? 1 Dysrhythmia 2 Alveolar collapse 3 Spinal cord injury 4 Impaired cardiac output

3 Injury to the phrenic nerve above C3 results in a complete spinal cord injury, causing complete diaphragm paralysis and dependence on a mechanical ventilator. Dysrhythmia, alveolar collapse, and impaired cardiac output are not causes of diaphragm paralysis that necessitate mechanical ventilation. Text Reference - p. 478

Which assessment finding of the respiratory system does the nurse interpret as normal? 1 Positive egophony 2 Fine crackles over the distal lung fields 3 Symmetric chest expansion and contraction 4 Bronchial breath sounds in the lower lung fields

3 Symmetric chest expansion and contraction is a normal assessment finding. Positive egophony and fine crackles are not normal findings. Bronchial or bronchovesicular sounds heard in the peripheral lung fields are abnormal breath sounds.

The nurse assesses a patient who presents with tachypnea and clubbing of the fingers. Which diagnosis does the nurse anticipate for this patient? 1 Asthma 2 Chest trauma 3 Chronic hypoxemia 4 Chronic pulmonary obstructive disease

3 Tachypnea and clubbing of the fingers support the diagnosis of chronic hypoxemia. Pursed-lip breathing, inability to lie in a flat position, and use of accessory muscles to assist with breathing are findings observed in patients with asthma and chronic obstructive pulmonary disease. Voluntary decrease in tidal volume to reduce pain on chest expansion is referred to as splinting, which is a common manifestation of chest trauma or pleurisy. Text Reference - p. 489

What is the major muscle of respiration? 1 Accessory muscle 2 Intercostal muscle 3 Diaphragm muscle 4 Abdominal muscle

3 The diaphragm is the major muscle of respiration. It is a sheet of internal skeletal muscle. An accessory muscle is a relatively rare anatomic duplication of muscle that may appear anywhere in the muscular system. The intercostal muscles are several groups of muscles that run between the ribs and help form and move the chest wall. The abdominal muscles support the trunk, allow movement, and hold organs in place by regulating internal abdominal pressure. They also assist in expelling air during labored breathing. Text Reference - p. 478

A patient has been admitted with a suspected lung abscess. During the assessment, the nurse is aware that the most common manifestation of a lung abscess is which of these? 1 Fever 2 Vomiting 3 Purulent sputum that has a foul odor and taste 4 Increased breath sounds on auscultation over the involved segment of lung.

3 The most common manifestation of a lung abscess is cough-producing purulent sputum (often dark brown) that is foul smelling and foul tasting. Hemoptysis is common, especially when an abscess ruptures into a bronchus. Other common manifestations are fever, chills, prostration, night sweats, pleuritic pain, dyspnea, anorexia, and weight loss. Physical examination of the lungs indicates dullness to percussion and decreased breath sounds on auscultation over the involved segment of lung. Vomiting is not a manifestation of a lung abscess. Text Reference - p. 534

The patient with human immunodeficiency virus (HIV) has been diagnosed with Candida albicans, an opportunistic infection. The nurse knows the patient needs more teaching when the patient says, 1 "I will be given amphotericin B to treat the fungus." 2 "I got this fungus because I am immunocompromised." 3 "I need to be isolated from my family and friends so they won't get it." 4 "The effectiveness of my therapy can be monitored with fungal serology titers."

3 The patient with an opportunistic fungal infection does not need to be isolated because C. albicans is not transmitted from person to person. This immunocompromised patient will be likely to have a serious infection so it will be treated with intravenous amphotericin B. The effectiveness of the therapy can be monitored with fungal serology titers.

The nurse is assessing a patient with chest trauma who is on chest-tube gravity drainage. While assessing, the nurse finds that the fluid level in the chamber is very high. What is the best nursing intervention in this situation? 1 Applying the clamp to the tube 2 Retaping the tube connections 3 Lowering the water seal column 4 Releasing the high negativity valve

4 A very high fluid level in the water chamber indicates high negative pressure in the patient with chest trauma. Therefore, the nurse should release the high negativity valve. Applying a clamp will stop the suction and decrease the risk of leakage but will not reduce negative pressure. Retaping the tube connections will reduce the leakage from the tube. The patient is on gravity drainage. Therefore, lowering the water seal column may cause complications for the patient.

The nurse is caring for a patient who is on a chest drainage tube with wet suction system. The nurse finds that the suction control chamber has no bubbles. What does the nurse expect from this finding? 1 The drainage system is blocked. 2 The chamber is getting evaporated. 3 The patient will have lung expansion. 4 The patient has a high pleural air leak.

4 Absence of bubbles indicates that the patient may have a high pleural air leak. Excessive bubbling in the water-seal chamber results in evaporation from the chamber. Tidaling in the water seal chamber indicates that the drainage system is blocked or the patient will have lung expansion due to suction. Text Reference - p. 546

What is the priority nursing intervention in helping a patient expectorate thick lung secretions? 1 Humidify the oxygen as able 2 Administer cough suppressant q4hr 3 Teach patient to splint the affected area 4 Increase fluid intake to 3 L/day if tolerated

4 Although several interventions may help the patient expectorate mucus, the highest priority should be on increasing fluid intake, which will liquefy the secretions so that the patient can expectorate them more easily. Humidifying the oxygen also is helpful, but is not the primary intervention. Teaching the patient to splint the affected area also may be helpful in decreasing discomfort, but does not assist in expectoration of thick secretions. Text Reference - p. 528 TERM ENGLISH

The nurse is reviewing the care of a patient with a pulmonary fungal infection. Which of these statements is true? 1 The patient will be placed on droplet isolation. 2 Opportunistic fungal infections occur in otherwise healthy people. 3 Pulmonary fungal infections are transmitted from person to person. 4 Amphotericin B is the standard therapy for treating serious systemic fungal infections.

4 Amphotericin B remains the standard therapy for treating serious systemic fungal infections. These infections are not transmitted from person to person, and the patient does not have to be placed in isolation. Opportunistic fungal infections occur in immunocompromised patients (e.g., those being treated with corticosteroids, chemotherapy, and immunosuppressive drugs) and in patients with human immunodeficiency virus (HIV) and cystic fibrosis. Fungal infections are acquired by inhalation of spores. Text Reference - p. 534

A 71-year-old patient is admitted with acute respiratory distress related to cor pulmonale. Which nursing intervention is most appropriate during admission of this patient? 1 Perform a comprehensive health history with the patient to review prior respiratory problems. 2 Complete a full physical examination to determine the effect of the respiratory distress on other body functions. 3 Delay any physical assessment of the patient and review with the family the patient's history of respiratory problems. 4 Perform a physical assessment of the respiratory system and ask specific questions related to this episode of respiratory distress.

4 Because the patient is having respiratory difficulty, the nurse should ask specific questions about this episode and perform a physical assessment of this system. Further history taking and physical examination of other body systems can proceed once the patient's acute respiratory distress is being managed.

A patient presents with a lung abscess. What treatment option would be the most appropriate? 1 Postural drainage 2 Chest physiotherapy 3 Reduction of fluid intake 4 Antibiotic treatment

4 Because there are mixed bacteria in a lung abscess, starting a broad spectrum antibiotic is the appropriate treatment option. Postural drainage and chest physiotherapy are not recommended, because they may cause spillage of infection to other bronchi and spread the infection. Reducing fluid intake is not advisable; instead, adequate fluid intake is recommended. Text Reference - p. 534

A patient is scheduled for a computed tomography (CT) scan with contrast medium. After reviewing the patient's laboratory reports, the nurse contacts the primary health care provider and the CT scan is cancelled. Which laboratory parameter would have made the diagnostic test unsafe for the patient? 1 Hematocrit 50% 2 PaCO2 40mm Hg 3 Hemoglobin 14.0 g/dL 4 Serum creatinine 3.0 mg/dL

4 CT scans may be often performed with contrast medium. These contrast media are excreted through urine; therefore, it is important for the patient to have optimal renal function to prevent accumulation of the contrast media in the body. The normal range of serum creatinine level is 0.6 to 1.3 mg/dL; therefore, a serum creatinine level of 3.0 mg/dL is very high and indicates renal dysfunction. As a result, the diagnostic test should not be performed on the patient. Hematocrit of 50%, PaCO2 of 40 mm Hg, and hemoglobin of 14.0 g/dL are within normal ranges. Text Reference - p. 491

The nurse is reviewing the laboratory reports for the patient receiving cefuroxime. The nurse determines that the medication is having the intended effect by noting which laboratory result? 1 Blood urea nitrogen (BUN) 8 mmol/L 2 Platelets 175,000 per µL 3 Sodium 138 mEq/L 4 White blood cell count 4500/mm

4 Depending on the source used, the normal white blood cell count is 4000 to 11,000/mm3or 5000 to 10,000/mm3. Because cefuroxime is a cephalosporin-type antibiotic, a white blood cell count within the normal range indicates resolution of infection. The BUN, platelet count, and sodium level are also normal; however, these results do not relate to antibiotic therapy. Text Reference - p. 525

When planning care for a patient with pneumonia, the nurse recognizes which intervention as the highest priority? 1 Administering analgesics as needed for pain 2 Keeping the patient in a calm and quiet environment 3 Routinely checking vital signs and oxygen saturation 4 Increasing fluids to 2 to 3 L/day unless contraindicated

4 Forcing fluids is a priority of care in patients with pneumonia. Fluids keep secretions liquefied and loose so they can be more easily raised and expectorated. Analgesics are important as is keeping the patient calm to decrease oxygen needs, but fluids, either IV or oral, are the first priority. Vital signs should be checked every four hours or more often as indicated. Text Reference - p. 528

A patient has just been admitted to the intensive care unit with a suspected diagnosis of pulmonary embolism (PE). The patient's condition is stable. The nurse will prepare for which intervention? 1 Oral administration of warfarin 2 Thrombolytic therapy with alteplase 3 Intravenous administration of unfractionated heparin 4 Subcutaneous administration of enoxaparin

4 Immediate anticoagulation is required for patients with PE. Subcutaneous administration of low-molecular-weight heparin (LMWH; e.g., enoxaparin) has been found to be safer and more effective than use of unfractionated heparin. It is the recommended choice of treatment for patients with nonmassive PE. Warfarin should be initiated within the first three days of heparinization and typically is administered for three to six months. Thrombolytic therapy in PE is done in cases of hemodynamic instability and right ventricular dysfunction. Text Reference - p. 553

A patient with a respiratory condition asks "I know how air gets into my lungs. But how does it get out of them?" The nurse bases the answer on the knowledge that air moves out of the lungs because of 1 Contraction of the accessory abdominal muscles 2 Increased carbon dioxide and decreased oxygen in the blood 3 Stimulation of the respiratory muscles by the chemoreceptors 4 Increase in intrathoracic pressure as elastic recoil of the lungs occurs

4 In contrast to inspiration, expiration is passive. Elastic recoil is the tendency for the lungs to relax after being stretched or expanded. The elasticity of lung tissue is due to the elastin fibers found in the alveolar walls and surrounding the bronchioles and capillaries. The elastic recoil of the chest wall and lungs allows the chest to passively decrease in volume. Intrathoracic pressure rises, causing air to move out of the lungs. Lungs do not expel air by use of abdominal muscles. An increase in carbon dioxide and decrease in oxygen in the blood do not promote expiration. Text Reference - p. 478

A patient with human immunodeficiency virus (HIV) infection has been admitted with pneumonia. The nurse anticipates that this patient will receive treatment for which type of pneumonia? 1 Aspiration pneumonia 2 Hospital-associated pneumonia 3 Community-acquired pneumonia 4 Opportunistic pneumonia caused by Pneumocystis jiroveci

4 Individuals at risk for opportunistic pneumonia include those with altered immune responses, such as HIV infection. In addition to the risk of bacterial and viral pneumonia, the immunocompromised person may develop an infection from microorganisms that do not normally cause disease, such as P.jiroveci (formerly carinii). The patient likely does not have aspiration pneumonia, hospital-associated pneumonia, or community-acquired pneumonia. Text Reference - p. 523

The nurse is caring for a patient with a non-massive pulmonary embolism (PE). What is the best standard for treatment? 1 Tissue plasminogen activator 2 Alteplase 3 Warfarin 4 Enoxaparin

4 Subcutaneous administration of low-molecular-weight heparin has been found to be safer and more effective than use of unfractionated heparin. It is also the recommended choice of treatment for patients with non-massive PE. Criteria for fibrinolytic therapy in PE include hemodynamic instability and right ventricular dysfunction. Fibrinolytic therapy dissolves pulmonary embolisms and the source of the thrombus. Low-molecular-weight heparin is becoming more common for non-massive pulmonary embolism. Warfarin should be initiated within the first 24 hours s of heparinization and is typically administered for three to six months. Warfarin therapy is the standard treatment for non-massive PE.

The nurse is measuring a patient's pulse oximetry and obtains an SpO2 reading of 88%, and then another reading of 87%. The patient is receiving oxygen per nasal cannula at 2 L/minute and states feeling short of breath. Previous readings were above 94%. What should the nurse do next? 1 Document the findings. 2 Increase the flow of oxygen delivery. 3 Check the pulse oximetry again in an hour. 4 Notify the health care provider for a possible arterial blood gas (ABG) analysis.

4 The abbreviation SpO2 is used to indicate the oxygen saturation of hemoglobin as measured by pulse oximetry. If there is doubt about the accuracy of the SpO2 reading, obtain an ABG analysis to verify the results. The findings should not be documented as normal. The nurse should not increase oxygen flow without further assessment.

What will be the immediate nursing action if the nurse, while caring for a patient with chest trauma, finds that the drainage system in the patient is broken? 1 Milking the drainage tubes 2 Stripping the drainage tubes 3 Emptying the collection chamber 4 Placing the chest tubing in a sterile water container

4 The drainage system should be properly checked to lessen the risk of complications. If the drainage system is found to be broken, then the distal end of the chest tubing connection should be placed in a sterile water container at a 2-cm level as an emergency water seal. Milking and stripping the drainage tubes are done only when there is an order from the physician. The collection chamber should never be emptied but should be replaced.

While palpating a patient's chest, the nurse places the palmar surface of the hands with hyperextended fingers against the patient's chest. The nurse asks the patient to say "ninety-nine" in a deeper, louder than normal voice. The nurse finds that the vibration of the sound is decreased in the patient. What does this finding indicate? 1 The patient has pneumonia 2 The patient has a lung tumor 3 The patient has pneumothorax 4 The patient has pleural effusion

4 The nurse assesses fremitus in a patient by palpating the chest with his or her hands in the manner described. If the patient has pleural effusion, there will be distance between the lung tissue and the nurse's hand, which will decrease the vibrations felt. Fremitus would be increased in cases of pneumonia and lung tumors, because the vibration of sound would be transmitted more through solid tissue. The nurse would not detect any fremitus in a patient with pneumothorax because the condition prevents any vibration from transmitting. Text Reference - p. 486

The nurse is caring for an older adult patient who presents with a cough, chest pain, shortness of breath, a heart rate of 110 beats/minute, a respiratory rate of 30 breaths/minute, and a temperature of 101° F. Based on these data, what does the nurse suspect is causing the patient's symptoms? 1 Hematemesis 2 Increased compliance 3 Alpha1-antitrypsin deficiency 4 Decreased alveolar macrophage function

4 There are certain age-related changes to the respiratory system that increase a patient's risk for developing respiratory disorders. The patient's symptoms suggest pneumonia. Based on these data, the nurse suspects that the current symptoms are caused by decreased alveolar macrophage functions, which are responsible for protecting the lungs against bacterial infection. Hematemesis, increased compliance, and an alpha1-antitrypsin deficiency are not the likely causes for this patient's symptoms. Text Reference - p. 481

The nurse provides preprocedure teaching for a patient who is scheduled for bedside thoracentesis. What does the nurse explain to be the primary purpose of thoracentesis? 1 Determining the stage of a lung tumor 2 Directly inspecting and examining the pleural space 3 Obtaining a specimen of pleural tissue for evaluation 4 Relieving an abnormal accumulation of fluid in the pleural space

4 Thoracentesis involves the insertion of a large-bore needle into the pleural space to relieve an abnormal accumulation of fluid in the pleural space. The procedure can significantly relieve symptoms related to this fluid accumulation, such as shortness of breath and discomfort. Thoracentesis cannot reveal the stage of lung cancer or permit direct inspection and examination of the pleural space. It may provide a pleural fluid specimen but not a pleural tissue specimen. Text Reference - p. 550

When the patient is experiencing metabolic acidosis secondary to type 1 diabetes mellitus, what physiologic response should the nurse expect to assess in the patient? 1 Vomiting 2 Increased urination 3 Decreased heart rate 4 Rapid respiratory rate

4 When a patient with type 1 diabetes has hyperglycemia and ketonemia causing metabolic acidosis, the physiologic response is to increase the respiratory rate and tidal volume to blow off the excess CO2. Vomiting and increased urination may occur with hyperglycemia, but not as physiologic responses to metabolic acidosis. The heart rate will increase. Text Reference - p. 481

Why does the primary health care provider, while managing a patient who is on chest drainage, clamp the chest tube a few hours before removal? 1 To minimize the risk of atelectasis 2 To reduce the risk of shoulder stiffness 3 To reduce the incidence of infected sites 4 To assess the patient's tolerance after removal

4 While planning to remove the chest drainage tube, the primary health care provider clamps the chest tube a few hours before removal to assess the patient's tolerance after removal. Adequate patient teaching is provided to minimize the risk of atelectasis and to reduce the risk of shoulder stiffness. Meticulous sterile technique during dressing changes is followed to reduce the incidence of infected sites. Text Reference - p. 547

When caring for elderly patients with hypertension, which information should the nurse consider when planning care (select all that apply.)? A. Systolic blood pressure increases with aging. B. Blood pressures should be maintained near 120/80 mm Hg. C. White coat syndrome is prevalent in elderly patients. D. Volume depletion contributes to orthostatic hypotension. E. Blood pressure drops 1 hour postprandially in many older patients. F. Older patients will require higher doses of antihypertensive medications.

A C D E Systolic blood pressure increases with age and patients older than age 60 years should be maintained below 150/90 mm Hg. Older patients have significantly higher blood pressure readings when taken by health care providers (white coat syndrome). Older patients experience orthostatic hypotension related to dehydration, reduced compensatory mechanisms, and medications. One hour after eating, many older patients experience a drop in blood pressure. Lower doses of medications may be needed to control blood pressures in older adults related to decreased absorption rates and excretion ability.

Despite a high dosage, a male patient who is taking nifedipine (Procardia XL) for antihypertensive therapy continues to have blood pressures over 140/90 mm Hg. What should the nurse do next? A. Assess his adherence to therapy. B. Ask him to make an exercise plan. C. Instruct him to use the DASH diet. D. Request a prescription for a thiazide diuretic.

A. A long-acting calcium-channel blocker such as nifedipine causes vascular smooth muscle relaxation, resulting in decreased systemic vascular resistance and arterial blood pressure and related side effects. The patient data the nurse has about this patient is very limited, so the nurse needs to begin by assessing adherence to therapy.

The nurse teaches a patient with hypertension that uncontrolled hypertension may damage organs in the body primarily by which mechanism? A. Hypertension promotes atherosclerosis and damage to the walls of the arteries. B. Hypertension causes direct pressure on organs, resulting in necrosis and replacement of cells with scar tissue. C. Hypertension causes thickening of the capillary membranes, leading to hypoxia of organ systems. D. Hypertension increases blood viscosity, which contributes to intravascular coagulation and tissue necrosis distal to occlusions.

A. Hypertension is a major risk factor for the development of atherosclerosis by mechanisms not yet fully known. However, when atherosclerosis develops, it damages the walls of arteries and reduces circulation to target organs and tissues.

The nurse is reviewing the laboratory test results for a 68-year-old patient whose warfarin (Coumadin) therapy was terminated during the preoperative period. The nurse concludes that the patient is in the most stable condition for surgery after noting which INR (international normalized ratio) result? A. 1.0 B. 1.8 C. 2.7 D. 3.4

A. 1.0 The therapeutic range for INR is 2.0 to 3.0 for many clinical diagnoses. The larger the INR number, the greater the amount of anticoagulation. For this reason, the safest value before surgery is 1.0, meaning that the anticoagulation has been reversed.

A 40-year-old man tells the nurse he has a diagnosis for the color and temperature changes of his limbs but can't remember the name of it. He says he must stop smoking and avoid trauma and exposure of his limbs to cold temperatures to get better. This description should allow the nurse to ask the patient if he has which diagnosis? A. Buerger's disease B. Venous thrombosis C. Acute arterial ischemia D. Raynaud's phenomenon

A. Buerger's disease Buerger's disease is a nonatherosclerotic, segmental, recurrent inflammatory disorder of small and medium-sized veins and arteries of upper and lower extremities leading to color and temperature changes of the limbs, intermittent claudication, rest pain, and ischemic ulcerations. It primarily occurs in men younger than 45 years old with a long history of tobacco and/or marijuana use. Buerger's disease treatment includes smoking cessation, trauma and cold temperature avoidance, and a walking program. Venous thrombosis is the formation of a thrombus in association with inflammation of the vein. Acute arterial ischemia is a sudden interruption in arterial blood flow to a tissue caused by embolism, thrombosis, or trauma. Raynaud's phenomenon is characterized by vasospasm-induced color changes of the fingers, toes, ears, and nose.

When the patient is being examined for venous thromboembolism (VTE) in the calf, what diagnostic test should the nurse expect to teach the patient about first? A. Duplex ultrasound B. Contrast venography C. Magnetic resonance venography D. Computed tomography venography

A. Duplex ultrasound The duplex ultrasound is the most widely used test to diagnose VTE. Contrast venography is rarely used now. Magnetic resonance venography is less accurate for calf veins than pelvic and proximal veins. Computed tomography venography may be used but is invasive and much more expensive than the duplex ultrasound.

What medications should the nurse expect to include in the teaching plan to decrease the risk of cardiovascular events and death for PAD patients (select all that apply)? A. Ramipril (Altace) B. Cilostazol (Pletal) C. Simvastatin (Zocor) D. Clopidogrel (Plavix) E. Warfarin (Coumadin) F. Aspirin (acetylsalicylic acid)

A. Ramipril (Altace) C. Simvastatin (Zocor) F. Aspirin (acetylsalicylic acid) Angiotensin-converting enzyme inhibitors (e.g., ramipril [Altace]) are used to control hypertension. Statins (e.g., simvastatin [Zocor]) are used for lipid management. Aspirin is used as an antiplatelet agent. Cilostazol (Pletal) is used for intermittent claudication, but it does not reduce CVD morbidity and mortality risks. Clopidogrel may be used if the patient cannot tolerate aspirin. Anticoagulants (e.g., warfarin [Coumadin]) are not recommended to prevent CVD events in PAD patients.

Assessment of a patient's peripheral IV site reveals that phlebitis has developed over the past several hours. Which intervention should the nurse implement first? A. Remove the patient's IV catheter. B. Apply an ice pack to the affected area. C. Decrease the IV rate to 20 to 30 mL/hr. D. Administer prophylactic anticoagulants.

A. Remove the patient's IV catheter. The priority intervention for superficial phlebitis is removal of the offending IV catheter. Decreasing the IV rate is insufficient. Anticoagulants are not normally required, and warm, moist heat is often therapeutic.

The patient has CVI and a venous ulcer. The unlicensed assistive personnel (UAP) decides to apply compression stockings because that is what these patients always have ordered. What assessment by the nurse would cause the application of compression stockings to harm the patient? A. Rest pain B. High blood pressure C. Elevated blood sugar D. Dry, itchy, flaky skin

A. Rest pain Rest pain occurs as peripheral artery disease (PAD) progresses and involves multiple arterial segments. Compression stockings should not be used on patients with PAD. Elevated blood glucose, possibly indicating uncontrolled diabetes mellitus, and hypertension may or may not indicate arterial problems. Dry, itchy, flaky skin indicates venous insufficiency. The RN should be the one to obtain the order and instruct the UAP to apply compression stockings if they are ordered.

The nurse is admitting a 68-year-old preoperative patient with a suspected abdominal aortic aneurysm (AAA). The medication history reveals that the patient has been taking warfarin (Coumadin) on a daily basis. Based on this history and the patient's admission diagnosis, the nurse should prepare to administer which medication? A. Vitamin K B. Cobalamin C. Heparin sodium D. Protamine sulfate

A. Vitamin K Coumadin is a Vitamin K antagonist anticoagulant that could cause excessive bleeding during surgery if clotting times are not corrected before surgery. For this reason, vitamin K is given as the antidote for warfarin (Coumadin).

A patient in the intensive care unit with acute decompensated heart failure (ADHF) complains of severe dyspnea and is anxious, tachypneic, and tachycardic. All of the following medications have been ordered for the patient. The nurse's priority action will be to a. give IV morphine sulfate 4 mg. b. give IV diazepam (Valium) 2.5 mg. c. increase nitroglycerin (Tridil) infusion by 5 mcg/min. d. increase dopamine (Intropin) infusion by 2 mcg/kg/min.

ANS: A Morphine improves alveolar gas exchange, improves cardiac output by reducing ventricular preload and afterload, decreases anxiety, and assists in reducing the subjective feeling of dyspnea. Diazepam may decrease patient anxiety, but it will not improve the cardiac output or gas exchange. Increasing the dopamine may improve cardiac output, but it will also increase the heart rate and myocardial oxygen consumption. Nitroglycerin will improve cardiac output and may be appropriate for this patient, but it will not directly reduce anxiety and will not act as quickly as morphine to decrease dyspnea.

After receiving change-of-shift report on a heart failure unit, which patient should the nurse assess first? a. A patient who is cool and clammy, with new-onset confusion and restlessness b. A patient who has crackles bilaterally in the lung bases and is receiving oxygen. c. A patient who had dizziness after receiving the first dose of captopril (Capoten) d. A patient who is receiving IV nesiritide (Natrecor) and has a blood pressure of 100/62

ANS: A The patient who has "wet-cold" clinical manifestations of heart failure is perfusing inadequately and needs rapid assessment and changes in management. The other patients also should be assessed as quickly as possible but do not have indications of severe decreases in tissue perfusion.

During a visit to a 78-year-old with chronic heart failure, the home care nurse finds that the patient has ankle edema, a 2-kg weight gain over the past 2 days, and complains of "feeling too tired to get out of bed." Based on these data, the best nursing diagnosis for the patient is a. activity intolerance related to fatigue. b. disturbed body image related to weight gain. c. impaired skin integrity related to ankle edema. d. impaired gas exchange related to dyspnea on exertion.

ANS: A The patient's statement supports the diagnosis of activity intolerance. There are no data to support the other diagnoses, although the nurse will need to assess for other patient problems.

Based on the Joint Commission Core Measures for patients with heart failure, which topics should the nurse include in the discharge teaching plan for a patient who has been hospitalized with chronic heart failure (select all that apply)? a. How to take and record daily weight b. Importance of limiting aerobic exercise c. Date and time of follow-up appointment d. Symptoms indicating worsening heart failure e. Actions and side effects of prescribed medications

ANS: A, C, D, E The Joint Commission Core Measures state that patients should be taught about prescribed medications, follow-up appointments, weight monitoring, and actions to take for worsening symptoms. Patients with heart failure are encouraged to begin or continue aerobic exercises such as walking, while self-monitoring to avoid excessive fatigue.

A 53-year-old patient with Stage D heart failure and type 2 diabetes asks the nurse whether heart transplant is a possible therapy. Which response by the nurse is most appropriate? a. "Because you have diabetes, you would not be a candidate for a heart transplant." b. "The choice of a patient for a heart transplant depends on many different factors." c. "Your heart failure has not reached the stage in which heart transplants are needed." d. "People who have heart transplants are at risk for multiple complications after surgery."

ANS: B Indications for a heart transplant include end-stage heart failure (Stage D), but other factors such as coping skills, family support, and patient motivation to follow the rigorous posttransplant regimen are also considered. Diabetic patients who have well-controlled blood glucose levels may be candidates for heart transplant. Although heart transplants can be associated with many complications, this response does not address the patient's question.

After receiving change-of-shift report on a heart failure unit, which patient should the nurse assess first? a. Patient who is taking carvedilol (Coreg) and has a heart rate of 58 b. Patient who is taking digoxin and has a potassium level of 3.1 mEq/L c. Patient who is taking isosorbide dinitrate/hydralazine (BiDil) and has a headache d. Patient who is taking captopril (Capoten) and has a frequent nonproductive cough

ANS: B The patient's low potassium level increases the risk for digoxin toxicity and potentially fatal dysrhythmias. The nurse should assess the patient for other signs of digoxin toxicity and then notify the health care provider about the potassium level. The other patients also have side effects of their medications, but their symptoms do not indicate potentially life-threatening complications.

A patient who is receiving dobutamine (Dobutrex) for the treatment of acute decompensated heart failure (ADHF) has the following nursing interventions included in the plan of care. Which action will be most appropriate for the registered nurse (RN) to delegate to an experienced licensed practical/vocational nurse (LPN/LVN)? a. Assess the IV insertion site for signs of extravasation. b. Teach the patient the reasons for remaining on bed rest. c. Monitor the patient's blood pressure and heart rate every hour. d. Titrate the rate to keep the systolic blood pressure >90 mm Hg.

ANS: C An experienced LPN/LVN would be able to monitor BP and heart rate and would know to report significant changes to the RN. Teaching patients, making adjustments to the drip rate for vasoactive medications, and monitoring for serious complications such as extravasation require RN level education and scope of practice.

Which diagnostic test will be most useful to the nurse in determining whether a patient admitted with acute shortness of breath has heart failure? a. Serum troponin b. Arterial blood gases c. B-type natriuretic peptide d. 12-lead electrocardiogram

ANS: C B-type natriuretic peptide (BNP) is secreted when ventricular pressures increase, as they do with heart failure. Elevated BNP indicates a probable or very probable diagnosis of heart failure. A twelve-lead electrocardiogram, arterial blood gases, and troponin may also be used in determining the causes or effects of heart failure but are not as clearly diagnostic of heart failure as BNP

A patient with heart failure has a new order for captopril (Capoten) 12.5 mg PO. After administering the first dose and teaching the patient about the drug, which statement by the patient indicates that teaching has been effective? a. "I will be sure to take the medication with food." b. "I will need to eat more potassium-rich foods in my diet." c. "I will call for help when I need to get up to use the bathroom." d. "I will expect to feel more short of breath for the next few days."

ANS: C Captopril can cause hypotension, especially after the initial dose, so it is important that the patient not get up out of bed without assistance until the nurse has had a chance to evaluate the effect of the first dose. The angiotensin-converting enzyme (ACE) inhibitors are potassium sparing, and the nurse should not teach the patient to purposely increase sources of dietary potassium. Increased shortness of breath is expected with the initiation of -adrenergic blocker therapy for heart failure, not for ACE inhibitor therapy. ACE inhibitors are best absorbed when taken an hour before eating.

A patient has recently started on digoxin (Lanoxin) in addition to furosemide (Lasix) and captopril (Capoten) for the management of heart failure. Which assessment finding by the home health nurse is a priority to communicate to the health care provider? a. Presence of 1 to 2+ edema in the feet and ankles b. Palpable liver edge 2 cm below the ribs on the right side c. Serum potassium level 3.0 mEq/L after 1 week of therapy d. Weight increase from 120 pounds to 122 pounds over 3 days

ANS: C Hypokalemia can predispose the patient to life-threatening dysrhythmias (e.g., premature ventricular contractions), and potentiate the actions of digoxin and increase the risk for digoxin toxicity, which can also cause life-threatening dysrhythmias. The other data indicate that the patient's heart failure requires more effective therapies, but they do not require nursing action as rapidly as the low serum potassium level.

The nurse plans discharge teaching for a patient with chronic heart failure who has prescriptions for digoxin (Lanoxin) and hydrochlorothiazide (HydroDIURIL). Appropriate instructions for the patient include a. limit dietary sources of potassium. b. take the hydrochlorothiazide before bedtime. c. notify the health care provider if nausea develops. d. skip the digoxin if the pulse is below 60 beats/minute.

ANS: C Nausea is an indication of digoxin toxicity and should be reported so that the provider can assess the patient for toxicity and adjust the digoxin dose, if necessary. The patient will need to include potassium-containing foods in the diet to avoid hypokalemia. Patients should be taught to check their pulse daily before taking the digoxin and if the pulse is less than 60, to call their provider before taking the digoxin. Diuretics should be taken early in the day to avoid sleep disruption.

A patient who has chronic heart failure tells the nurse, "I was fine when I went to bed, but I woke up in the middle of the night feeling like I was suffocating!" The nurse will document this assessment finding as a. orthopnea. b. pulsus alternans. c. paroxysmal nocturnal dyspnea. d. acute bilateral pleural effusion.

ANS: C Paroxysmal nocturnal dyspnea is caused by the reabsorption of fluid from dependent body areas when the patient is sleeping and is characterized by waking up suddenly with the feeling of suffocation. Pulsus alternans is the alternation of strong and weak peripheral pulses during palpation. Orthopnea indicates that the patient is unable to lie flat because of dyspnea. Pleural effusions develop over a longer time period.

IV sodium nitroprusside (Nipride) is ordered for a patient with acute pulmonary edema. During the first hours of administration, the nurse will need to titrate the nitroprusside rate if the patient develops a. ventricular ectopy. b. a dry, hacking cough. c. a systolic BP <90 mm Hg. d. a heart rate <50 beats/minute.

ANS: C Sodium nitroprusside is a potent vasodilator, and the major adverse effect is severe hypotension. Coughing and bradycardia are not adverse effects of this medication. Nitroprusside does not cause increased ventricular ectopy.

The nurse working on the heart failure unit knows that teaching an older female patient with newly diagnosed heart failure is effective when the patient states that a. she will take furosemide (Lasix) every day at bedtime. b. the nitroglycerin patch is applied when any chest pain develops. c. she will call the clinic if her weight goes from 124 to 128 pounds in a week. d. an additional pillow can help her sleep if she is feeling short of breath at night.

ANS: C Teaching for a patient with heart failure includes information about the need to weigh daily and notify the health care provider about an increase of 3 pounds in 2 days or 3 to 5 pounds in a week. Nitroglycerin patches are used primarily to reduce preload (not to prevent chest pain) in patients with heart failure and should be used daily, not on an "as needed" basis. Diuretics should be taken earlier in the day to avoid nocturia and sleep disturbance. The patient should call the clinic if increased orthopnea develops, rather than just compensating by further elevating the head of the bed.

A patient with chronic heart failure who is taking a diuretic and an angiotensin-converting enzyme (ACE) inhibitor and who is on a low-sodium diet tells the home health nurse about a 5-pound weight gain in the last 3 days. The nurse's priority action will be to a. have the patient recall the dietary intake for the last 3 days. b. ask the patient about the use of the prescribed medications. c. assess the patient for clinical manifestations of acute heart failure. d. teach the patient about the importance of restricting dietary sodium.

ANS: C The 5-pound weight gain over 3 days indicates that the patient's chronic heart failure may be worsening. It is important that the patient be assessed immediately for other clinical manifestations of decompensation, such as lung crackles. A dietary recall to detect hidden sodium in the diet, reinforcement of sodium restrictions, and assessment of medication compliance may be appropriate interventions but are not the first nursing actions indicated.

Which topic will the nurse plan to include in discharge teaching for a patient with systolic heart failure and an ejection fraction of 33%? a. Need to begin an aerobic exercise program several times weekly b. Use of salt substitutes to replace table salt when cooking and at the table c. Benefits and side effects of angiotensin-converting enzyme (ACE) inhibitors d. Importance of making an annual appointment with the primary care provider

ANS: C The core measures for the treatment of heart failure established by The Joint Commission indicate that patients with an ejection fraction (EF) <40% receive an ACE inhibitor to decrease the progression of heart failure. Aerobic exercise may not be appropriate for a patient with this level of heart failure, salt substitutes are not usually recommended because of the risk of hyperkalemia, and the patient will need to see the primary care provider more frequently than annually

While admitting an 82-year-old with acute decompensated heart failure to the hospital, the nurse learns that the patient lives alone and sometimes confuses the "water pill" with the "heart pill." When planning for the patient's discharge the nurse will facilitate a a. consult with a psychologist. b. transfer to a long-term care facility. c. referral to a home health care agency. d. arrangements for around-the-clock care.

ANS: C The data about the patient suggest that assistance in developing a system for taking medications correctly at home is needed. A home health nurse will assess the patient's home situation and help the patient develop a method for taking the two medications as directed. There is no evidence that the patient requires services such as a psychologist consult, long-term care, or around-the-clock home care.

While assessing a 68-year-old with ascites, the nurse also notes jugular venous distention (JVD) with the head of the patient's bed elevated 45 degrees. The nurse knows this finding indicates a. decreased fluid volume. b. jugular vein atherosclerosis. c. increased right atrial pressure. d. incompetent jugular vein valves.

ANS: C The jugular veins empty into the superior vena cava and then into the right atrium, so JVD with the patient sitting at a 45-degree angle reflects increased right atrial pressure. JVD is an indicator of excessive fluid volume (increased preload), not decreased fluid volume. JVD is not caused by incompetent jugular vein valves or atherosclerosis.

An outpatient who has chronic heart failure returns to the clinic after 2 weeks of therapy with metoprolol (Toprol XL). Which assessment finding is most important for the nurse to report to the health care provider? a. 2+ pedal edema b. Heart rate of 56 beats/minute c. Blood pressure (BP) of 88/42 mm Hg d. Complaints of fatigue

ANS: C The patient's BP indicates that the dose of metoprolol may need to be decreased because of hypotension. Bradycardia is a frequent adverse effect of -adrenergic blockade, but the rate of 56 is not unusual with â-adrenergic blocker therapy. -Adrenergic blockade initially will worsen symptoms of heart failure in many patients, and patients should be taught that some increase in symptoms, such as fatigue and edema, is expected during the initiation of therapy with this class of drugs

A patient with a history of chronic heart failure is admitted to the emergency department (ED) with severe dyspnea and a dry, hacking cough. Which action should the nurse do first? a. Auscultate the abdomen. b. Check the capillary refill. c. Auscultate the breath sounds. d. Assess the level of orientation.

ANS: C This patient's severe dyspnea and cough indicate that acute decompensated heart failure (ADHF) is occurring. ADHF usually manifests as pulmonary edema, which should be detected and treated immediately to prevent ongoing hypoxemia and cardiac/respiratory arrest. The other assessments will provide useful data about the patient's volume status and also should be accomplished rapidly, but detection (and treatment) of pulmonary complications is the priority.

Following an acute myocardial infarction, a previously healthy 63-year-old develops clinical manifestations of heart failure. The nurse anticipates discharge teaching will include information about a. digitalis preparations. b. -adrenergic blockers. c. calcium channel blockers. d. angiotensin-converting enzyme (ACE) inhibitors.

ANS: D ACE inhibitor therapy is currently recommended to prevent the development of heart failure in patients who have had a myocardial infarction and as a first-line therapy for patients with chronic heart failure. Digoxin therapy for heart failure is no longer considered a first-line measure, and digoxin is added to the treatment protocol when therapy with other medications such as ACE-inhibitors, diuretics, and -adrenergic blockers is insufficient. Calcium channel blockers are not generally used in the treatment of heart failure. The -adrenergic blockers are not used as initial therapy for new onset heart failure.

A patient who has just been admitted with pulmonary edema is scheduled to receive the following medications. Which medication should the nurse question before giving? a. Furosemide (Lasix) 60 mg b. Captopril (Capoten) 25 mg c. Digoxin (Lanoxin) 0.125 mg d. Carvedilol (Coreg) 3.125 mg

ANS: D Although carvedilol is appropriate for the treatment of chronic heart failure, it is not used for patients with acute decompensated heart failure (ADHF) because of the risk of worsening the heart failure. The other medications are appropriate for the patient with ADHF.

The nurse is caring for a patient who is receiving IV furosemide (Lasix) and morphine for the treatment of acute decompensated heart failure (ADHF) with severe orthopnea. Which clinical finding is the best indicator that the treatment has been effective? a. Weight loss of 2 pounds in 24 hours b. Hourly urine output greater than 60 mL c. Reduction in patient complaints of chest pain d. Reduced dyspnea with the head of bed at 30 degrees

ANS: D Because the patient's major clinical manifestation of ADHF is orthopnea (caused by the presence of fluid in the alveoli), the best indicator that the medications are effective is a decrease in dyspnea with the head of the bed at 30 degrees. The other assessment data also may indicate that diuresis or improvement in cardiac output has occurred, but are not as specific to evaluating this patient's response.

When teaching the patient with newly diagnosed heart failure about a 2000-mg sodium diet, the nurse explains that foods to be restricted include a. canned and frozen fruits. b. fresh or frozen vegetables. c. eggs and other high-protein foods. d. milk, yogurt, and other milk products.

ANS: D Milk and yogurt naturally contain a significant amount of sodium, and intake of these should be limited for patients on a diet that limits sodium to 2000 mg daily. Other milk products, such as processed cheeses, have very high levels of sodium and are not appropriate for a 2000-mg sodium diet. The other foods listed have minimal levels of sodium and can be eaten without restriction.

The patient has acute bronchitis. What is the most important factor for the nurse to consider for this patient? 1 Clinical assessment finding of egophony 2 Presence of tactile fremitus 3 Therapy is mainly supportive 4 Abnormal chest x-ray

Acute bronchitis is usually self-limiting and treatment for acute bronchitis is supportive. Chest x-rays will differentiate acute bronchitis form pneumonia. In bronchitis, there is not consolidation or infiltrates on x-ray as there is in pneumonia. Text Reference - p. 5493

Palliative treatment for airway collapse or external compression

Airway stenting

The nurse admits a 73-yr-old male patient with dementia for treatment of uncontrolled hypertension. The nurse will closely monitor for hypokalemia if the patient receives which medication? A. Clonidine (Catapres) B. Bumetanide (Bumex) C.Amiloride (Midamor) D. Spironolactone (Aldactone)

B. Bumetanide is a loop diuretic. Hypokalemia is a common adverse effect of this medication. Amiloride is a potassium-sparing diuretic. Spironolactone is an aldosterone-receptor blocker. Hyperkalemia is an adverse effect of both amiloride and spironolactone. Clonidine is a central-acting α-adrenergic antagonist and does not cause electrolyte abnormalities.

The nurse is caring for a patient admitted with a history of hypertension. The patient's medication history includes hydrochlorothiazide daily for the past 10 years. Which parameter would indicate the optimal intended effect of this drug therapy? A. Weight loss of 2 lb B. BP 128/86 mm Hg Correct C. Absence of ankle edema D. Output of 600 mL per 8 hours

B. Hydrochlorothiazide may be used alone as monotherapy to manage hypertension or in combination with other medications if not effective alone. After the first few weeks of therapy, the diuretic effect diminishes, but the antihypertensive effect remains. Because the patient has been taking this medication for 10 years, the most direct measurement of its intended effect would be the blood pressure.

When providing dietary instruction to a patient with hypertension, the nurse would advise the patient to restrict intake of which meat? A. Broiled fish B. Roasted duck Correct C. Roasted turkey D. Baked chicken breast

B. Roasted duck is high in fat, which should be avoided by the patient with hypertension. Weight loss may slow the progress of atherosclerosis and overall cardiovascular disease risk. The other meats are lower in fat and are therefore acceptable in the diet.

The nurse supervises an unlicensed assistant personnel (UAP) who is taking the blood pressure of 58-yr-old obese female patient admitted with heart failure. Which action by the UAP will require the nurse to intervene? A. Waiting 2 minutes after position changes to take orthostatic pressures B. Deflating the blood pressure cuff at a rate of 8 to 10 mm Hg per second C. Taking the blood pressure with the patient's arm at the level of the heart D. Taking a forearm blood pressure because the largest cuff will not fit the patient's upper arm

B. The cuff should be deflated at a rate of 2 to 3 mm Hg per second. The arm should be supported at the level of the heart for accurate blood pressure measurements. Using a cuff that is too small causes a falsely high reading and too large causes a falsely low reading. If the maximum size blood pressure cuff does not fit the upper arm, the forearm may be used. Orthostatic blood pressures should be taken within 1 to 2 minutes of repositioning the patient.

In caring for a patient admitted with poorly controlled hypertension, which laboratory test result should the nurse understand as indicating the presence of target organ damage? A. Serum uric acid of 3.8 mg/dL B. Serum creatinine of 2.6 mg/dL C. Serum potassium of 3.5 mEq/L D. Blood urea nitrogen of 15 mg/dL

B. The normal serum creatinine level is 0.6 to 1.3 mg/dL. This elevated level indicates target organ damage to the kidneys. The other laboratory results are within normal limits.

When teaching a patient about dietary management of stage 1 hypertension, which instruction is most appropriate? A. Increase water intake. B. Restrict sodium intake. C. Increase protein intake. D. Use calcium supplements.

B. The patient should decrease intake of sodium. This will help to control hypertension, which can be aggravated by excessive salt intake, which in turn leads to fluid retention. Protein intake does not affect hypertension. Calcium supplements are not recommended to lower blood pressure.

A patient with varicose veins has been prescribed compression stockings. How should the nurse teach the patient to use these? A. "Try to keep your stockings on 24 hours a day, as much as possible." B. "While you're still lying in bed in the morning, put on your stockings." C. "Dangle your feet at your bedside for 5 minutes before putting on your stockings." D. "Your stockings will be most effective if you can remove them for a few minutes several times a day."

B. "While you're still lying in bed in the morning, put on your stockings." The patient with varicose veins should apply stockings in bed, before rising in the morning. Stockings should not be worn continuously, but they should not be removed several times daily. Dangling at the bedside prior to application is likely to decrease their effectiveness.

A nurse is caring for a patient with a diagnosis of deep venous thrombosis (DVT). The patient has an order to receive 30 mg enoxaparin (Lovenox). Which injection site should the nurse use to administer this medication safely? A. Buttock, upper outer quadrant B. Abdomen, anterior-lateral aspect C. Back of the arm, 2 inches away from a mole D. Anterolateral thigh, with no scar tissue nearby

B. Abdomen, anterior-lateral aspect Enoxaparin (Lovenox) is a low-molecular-weight (LMW) heparin that is given as a deep subcutaneous injection in the right and left anterolateral abdomen. All subcutaneous injections should be given away from scars, lesions, or moles.

A female patient with critical limb ischemia has had peripheral artery bypass surgery to improve her circulation. What care should the nurse provide on postoperative day 1? A. Keep the patient on bed rest. B. Assist the patient with walking several times. C. Have the patient sit in the chair several times. D. Place the patient on her side with knees flexed.

B. Assist the patient with walking several times. To avoid blockage of the graft or stent, the patient should walk several times on postoperative day 1 and subsequent days. Having the patient's knees flexed for sitting in a chair or in bed increase the risk of venous thrombosis and may place stress on the suture lines.

The nurse is caring for a newly admitted patient with vascular insufficiency. The patient has a new order for enoxaparin (Lovenox) 30 mg subcutaneously. What should the nurse do to correctly administer this medication? A. Spread the skin before inserting the needle. B. Leave the air bubble in the prefilled syringe. C. Use the back of the arm as the preferred site. D. Sit the patient at a 30-degree angle before administration.

B. Leave the air bubble in the prefilled syringe. The nurse should not expel the air bubble from the prefilled syringe because it should be injected to clear the needle of medication and avoid leaving medication in the needle track in the tissue.

A patient was just diagnosed with acute arterial ischemia in the left leg secondary to atrial fibrillation. Which early clinical manifestation must be reported to the physician immediately to save the patient's limb? A. Paralysis B. Paresthesia C. Crampiness D. Referred pain

B. Paresthesia The physician must be notified immediately if any of the six Ps of acute arterial ischemia occur to prevent ischemia from quickly progressing to tissue necrosis and gangrene. The six Ps are paresthesia, pain, pallor, pulselessness, and poikilothermia, with paralysis being a very late sign indicating the death of nerves to the extremity. Crampy leg sensation is more common with varicose veins. The pain is not referred.

A 62-year-old Hispanic male patient with diabetes mellitus has been diagnosed with peripheral artery disease (PAD). The patient is a smoker and has a history of gout. What should the nurse focus her teaching on to prevent complications for this patient? A. Gender B. Smoking C. Ethnicity D. Co-morbidities

B. Smoking Smoking is the most significant factor for this patient. PAD is a marker of advanced systemic atherosclerosis. Therefore tobacco cessation is essential to reduce PAD progression, CVD events, and mortality. Diabetes mellitus and hyperuricemia are also risk factors. Being male or Hispanic are not risk factors for PAD.

The patient had aortic aneurysm repair. What priority nursing action will the nurse use to maintain graft patency? A. Assess output for renal dysfunction. B. Use IV fluids to maintain adequate BP. C. Use oral antihypertensives to maintain cardiac output. D. Maintain a low BP to prevent pressure on surgical site

B. Use IV fluids to maintain adequate BP. The priority is to maintain an adequate BP (determined by the surgeon) to maintain graft patency. A prolonged low BP may result in graft thrombosis, and hypertension may cause undue stress on arterial anastomoses resulting in leakage of blood or rupture at the suture lines, which is when IV antihypertensives may be used. Renal output will be assessed when the aneurysm repair is above the renal arteries to assess graft patency, not maintain it.

Palliative treatment by bronchoscope to remove obstructing bronchial tumors

Bronchoscopic laser

The nurse teaches a 28-yr-old man newly diagnosed with hypertension about lifestyle modifications to reduce his blood pressure. Which patient statement requires reinforcement of teaching? A. "I will avoid adding salt to my food during or after cooking." B."If I lose weight, I might not need to continue taking medications." C. "I can lower my blood pressure by switching to smokeless tobacco." D. "Diet changes can be as effective as taking blood pressure medications.

C Nicotine contained in tobacco products (smoking and chew) cause vasoconstriction and increase blood pressure. Persons with hypertension should restrict sodium to 1500 mg/day by avoiding foods high in sodium and not adding salt in preparation of food or at meals. Weight loss can decrease blood pressure between 5 to 20 mm Hg. Following dietary recommendations (e.g., the DASH diet) lowers blood pressure, and these decreases compare with those achieved with blood pressure-lowering medication.

The nurse is teaching a women's group about prevention of hypertension. What information should be included in the teaching for all the women (select all that apply.)? A. Lose weight. B. Limit nuts and seeds. C. Limit sodium and fat intake. D. Increase fruits and vegetables. E. Exercise 30 minutes most days.

C D E Primary prevention of hypertension is to make lifestyle modifications that prevent or delay the increase in BP. Along with exercise for 30 minutes on most days, the DASH eating plan is a healthy way to lower BP by limiting sodium and fat intake, increasing fruits and vegetables, and increasing nutrients that are associated with lowering BP. Nuts and seeds and dried beans are used for protein intake. Weight loss may or may not be necessary for the individual.

The nurse is caring for a patient admitted with chronic obstructive pulmonary disease (COPD), angina, and hypertension. Before administering the prescribed daily dose of atenolol 100 mg PO, the nurse assesses the patient carefully. Which adverse effect is this patient at risk for given the patient's health history? A. Hypocapnia B. Tachycardia C. Bronchospasm D. Nausea and vomiting

C. Atenolol is a cardioselective β1-adrenergic blocker that reduces blood pressure and could affect the β2-receptors in the lungs with larger doses or with drug accumulation. Although the risk of bronchospasm is less with cardioselective β-blockers than nonselective β-blockers, atenolol should be used cautiously in patients with COPD.

The UAP is taking orthostatic vital signs. In the supine position, the blood pressure (BP) is 130/80 mm Hg, and the heart rate (HR) is 80 beats/min. In the sitting position, the BP is 140/80, and the HR is 90 beats/min. Which action should the nurse instruct the UAP to take next? A. Repeat BP and HR in this position. B. Record the BP and HR measurements. C. Take BP and HR with patient standing. D. Return the patient to the supine position

C. The vital signs taken do not reflect orthostatic changes, so the UAP will continue with the measurements while the patient is standing. There is no need to repeat or delay the readings. The patient does not need to return to the supine positon. When assessing for orthostatic changes, the UAP will take the BP and pulse in the supine position, then place the patient in a sitting position for 1 to 2 minutes and repeat the readings, and then reposition to the standing position for 1 to 2 minutes and repeat the readings. Results consistent with orthostatic changes would have a decrease of 20 mm Hg or more in systolic BP, a decrease of 10 mm Hg or more in diastolic BP, and/or an increase in HR of greater than or equal to 20 beats/min with position changes.

A postoperative patient asks the nurse why the physician ordered daily administration of enoxaparin (Lovenox). Which reply by the nurse is most appropriate? A. "This medication will help prevent breathing problems after surgery, such as pneumonia." B. "This medication will help lower your blood pressure to a safer level, which is very important after surgery." C. "This medication will help prevent blood clots from forming in your legs until your level of activity, such as walking, returns to normal." D. "This medication is a narcotic pain medication that will help take away any muscle aches caused by positioning on the operating room table."

C. "This medication will help prevent blood clots from forming in your legs until your level of activity, such as walking, returns to normal." Enoxaparin is an anticoagulant that is used to prevent DVTs postoperatively. All other explanations/options do not describe the action/purpose of enoxaparin.

The nurse is caring for a patient who has been receiving warfarin (Coumadin) and digoxin (Lanoxin) as treatment for atrial fibrillation. Because the warfarin has been discontinued before surgery, the nurse should diligently assess the patient for which complication early in the postoperative period until the medication is resumed? A. Decreased cardiac output B. Increased blood pressure C. Cerebral or pulmonary emboli D. Excessive bleeding from incision or IV sites

C. Cerebral or pulmonary emboli Warfarin is an anticoagulant that is used to prevent thrombi from forming on the walls of the atria during atrial fibrillation. Once the medication is terminated, thrombi could again form. If one or more thrombi detach from the atrial wall, they could travel as cerebral emboli from the left atrium or pulmonary emboli from the right atrium.

The nurse would determine that a postoperative patient is not receiving the beneficial effects of enoxaparin (Lovenox) after noting what during a routine shift assessment? A. Generalized weakness and fatigue B. Crackles bilaterally in the lung bases C. Pain and swelling in lower extremity D. Abdominal pain with decreased bowel sounds

C. Pain and swelling in lower extremity Enoxaparin is a low-molecular-weight heparin used to prevent the development of deep vein thromboses (DVTs) in the postoperative period. Pain and swelling in the lower extremity can indicate development of DVT and therefore may signal ineffective medication therapy.

The nurse is caring for a preoperative patient who has an order for vitamin K by subcutaneous injection. The nurse should verify that which laboratory study is abnormal before administering the dose? A. Hematocrit (Hct) B. Hemoglobin (Hgb) C. Prothrombin time (PT) D. Partial thromboplastin time (PTT)

C. Prothrombin time (PT) Vitamin K counteracts hypoprothrombinemia and/or reverses the effects of warfarin (Coumadin) and thus decreases the risk of bleeding. High values for either the prothrombin time (PT) or the international normalized ratio (INR) demonstrates the need for this medication.

Describe the function of each chamber. Chamber Function Water-seal Suction control Collection Suction monitor bellows

Chamber Function Water-seal This chamber contains 2 cm of water, which acts as a one-way valve. Incoming air enters from the collection chamber and bubbles up through the water. The water prevents backflow of the air into the patient from the system Suction control This chamber supplies suction to the chest drainage. The water suction type system contains a column of water with the top end vented to the atmosphere to control the amount of suction, with bubbles to indicate it is working. The amount of suction applied is controlled by the amount of water in the chamber (usually −20 cm), not by the wall suction applied to it. The dry suction device contains no water and uses a regulator to dial the desired negative pressure. Collection This chamber receives fluid and air from the pleural or mediastinal space. Nurses keep track of the amount of drainage and can mark the container for easy measuring. Suction monitor bellows The dry model has the suction monitor bellows, which expands to show that the suction is operating, as this model does not make the bubbling noise made by the models that control

A 67-yr-old woman with hypertension is admitted to the emergency department with a blood pressure of 234/148 mm Hg and was started on nitroprusside (Nitropress). After one hour of treatment, the mean arterial blood pressure (MAP) is 55 mm Hg. Which nursing action is a priority? A. Start an infusion of 0.9% normal saline at 100 mL/hr. B. Maintain the current administration rate of the nitroprusside. C. Request insertion of an arterial line for accurate blood pressure monitoring. D. Stop the nitroprusside infusion and assess the patient for potential complications.

D. Nitroprusside is a potent vasodilator medication. A blood pressure of 234/118 mm Hg would have a calculated MAP of 177 mm Hg. Subtracting 25% (or 44 mm Hg) = 133 mm Hg. The initial treatment goal is to decrease MAP by no more than 25% within minutes to 1 hour. For this patient, the goal MAP would be approximately 133 mm Hg. Minimal MAP required to perfuse organs is around 60 to 65 mm Hg. Lowering the blood pressure too rapidly may decrease cerebral, coronary, or renal perfusion and could precipitate a stroke, myocardial infarction, or renal failure. The priority is to stop the nitroprusside infusion and then use fluids only if necessary to support restoration of MAP.

The nurse is caring for a patient with a recent history of deep vein thrombosis (DVT). The patient now needs to undergo surgery for appendicitis. The nurse is reviewing the laboratory results for this patient before administering an ordered dose of vitamin K. The nurse determines that the medication is both safe to give and is most needed when the international normalized ratio (INR) is which result? A. 1.0 B. 1.2 C. 1.6 D. 2.2

D. 2.2 Vitamin K is the antidote to warfarin (Coumadin), which the patient has most likely been taking before admission for treatment of DVT. Warfarin is an anticoagulant that impairs the ability of the blood to clot. Therefore it is necessary to give vitamin K before surgery to reduce the risk of hemorrhage. The largest value of the INR indicates the greatest impairment of clotting ability, making 2.2 the correct selection.

A male patient was admitted for a possible ruptured aortic aneurysm, but had no back pain. Ten minutes later his assessment includes the following: sinus tachycardia at 138, BP palpable at 65 mm Hg, increasing waist circumference, and no urine output. How should the nurse interpret this assessment about the patient's aneurysm? A. Tamponade will soon occur. B. The renal arteries are involved. C. Perfusion to the legs is impaired. D. He is bleeding into the abdomen.

D. He is bleeding into the abdomen. The lack of back pain indicates the patient is most likely exsanguinating into the abdominal space, and the bleeding is likely to continue without surgical repair. A blockade of the blood flow will not occur in the abdominal space as it would in the retroperitoneal space where surrounding anatomic structures may control the bleeding. The lack of urine output does not indicate renal artery involvement, but that the bleeding is occurring above the renal arteries, which decreases the blood flow to the kidneys. There is no assessment data indicating decreased perfusion to the legs.

The nurse is preparing to administer a scheduled dose of enoxaparin (Lovenox) 30 mg subcutaneously. What should the nurse do to administer this medication correctly? A. Remove the air bubble in the prefilled syringe. B. Aspirate before injection to prevent IV administration. C. Rub the injection site after administration to enhance absorption. D. Pinch the skin between the thumb and forefinger before inserting the needle.

D. Pinch the skin between the thumb and forefinger before inserting the needle. The nurse should gather together or "bunch up" the skin between the thumb and the forefinger before inserting the needle into the subcutaneous tissue. The nurse should not remove the air bubble in the prefilled syringe, aspirate, nor rub the site after injection.

TB patients' compliance with drug therapy has been poor and there has been general decreased vigilance in monitoring and follow-up of TB treatment causing...

Drug-resistant strains of TB to develope

late symptoms of TB are

Dyspnea and hemoptysis

acute sudden presentation of TB may occur with

Pleuritic pain, flu-like symptoms, and a productive cough

HAI pneumonia is frequently caused by

Pseudomonas aeruginosa, Escherichia coli

A patient with chronic HF and atrial fibrillation is treated with a digitalis glycoside and a loop diuretic. To prevent possible complications of this combination of drugs, what does the nurse need to do (select all that apply)? a. Monitor serum potassium levels b. teach the patient how to take a pulse rate. c. keep an accurate measure of intake and output d. Teach the patient about dietary restriction or potassium e. Withhold digitalis and notify health care provider if heart rate is irregular

a & b Rationale: Hypokalemia, which can be caused by the use of potassium-depleting diuretics (e.g., thiazides, loop diuretics), is one of the most common causes of digitalis toxicity. Low serum levels of potassium enhance the action of digitalis, causing a therapeutic dose to achieve toxic levels. Hypokalemia can also precipitate dysrhythmias. Monitoring the serum potassium levels of patients receiving digitalis preparations and potassium-depleting diuretics is essential. Patients taking digitalis preparations should be taught how to measure their pulse rate because bradycardia and atrioventricular blocks are late signs of digitalis toxicity. In addition, patients should know what pulse rate would necessitate a call to the health care provider.

With removal of the larynx, the patient will not be able to communicate verbally and it is important to arrange with the patient

a method of communication before surgery so that postoperative communication can take place.

Percussion dullness on the injured site indicates the presence of blood or fluid and decreased movement and diminished breath sounds are characteristic of

a pneumothorax.

A patient admitted with heart failure appears very anxious and complains of shortness of breath. Which nursing actions would be appropriate to alleviate this patient's anxiety (select all that apply)? a. Administer ordered morphine sulfate. b. Position patient in a semi-Fowler's position. c. Position patient on left side with head of bed flat. d. Instruct patient on the use of relaxation techniques. e. Use a calm, reassuring approach while talking to patient.

a, b, d, & e Morphine sulfate reduces anxiety and may assist in reducing dyspnea. The patient should be positioned in semi-Fowler's position to improve ventilation that will reduce anxiety. Relaxation techniques and a calm reassuring approach will also serve to reduce anxiety.

The patient has heart failure (HF) with an ejection fraction of less than 40%. What core measures should the nurse expect to include in the plan of care for this patient (select all that apply)? a. Left ventricular function is documented. b. Controlling dysrhythmias will eliminate HF. c. Prescription for digoxin (Lanoxin) at discharge d. Prescription for angiotensin-converting enzyme (ACE) inhibitor at discharge e. Education materials about activity, medications, weight monitoring, and what to do if symptoms worsen

a, d, & e The Joint Commission has identified these three core measures for heart failure patients. Although controlling dysrhythmias will improve CO and workload, it will not eliminate HF. Prescribing digoxin for all HF patients is no longer done because there are newer effective drugs and digoxin toxicity occurs easily related to electrolyte levels and the therapeutic range must be maintained.

A 36-year-old patient with type 1 diabetes mellitus asks the nurse whether an influenza vaccine is necessary every year. What is the best response by the nurse? a. "You should get the trivalent inactivated influenza vaccine that is injected every year." b. "Only health care workers in contact with high-risk patients should be immunized each year." c. "An annual vaccination is not necessary because previous immunity will protect you for several years." d. "Antiviral drugs, such as zanamivir (Relenza), eliminate the need for vaccine except in the older adult."

a. "You should get the trivalent inactivated influenza vaccine that is injected every year."

How do microorganisms reach the lungs and cause pneumonia? (select all that apply) a. Aspiration b. Lymphatic spread c. Inhalation of microbes in the air d. Touch contact with the infectious microbes e. Hematogenous spread from infections elsewhere in the body

a. Aspiration c. Inhalation of microbes in the air e. Hematogenous spread from infections elsewhere in the body

Priority Decision: A patient's tracheostomy tube becomes dislodged with vigorous coughing. What should be the nurse's first action? a. Attempt to replace the tube. b. Notify the health care provider. c. Place the patient in high Fowler position. d. Ventilate the patient with a manual resuscitation bag until the health care provider arrives

a. Attempt to replace the tube.

When obtaining a health history from a 76-year-old patient with suspected CAP, what does the nurse expect the patient or caregiver to report? a. Confusion c. An abrupt onset of fever and chills b. A recent loss of consciousness d. A gradual onset of headache and sore throat

a. Confusion

Priority Decision: Two days after undergoing pelvic surgery, a patient develops marked dyspnea and anxiety. What is the first action that the nurse should take? a. Raise the head of the bed. b. Notify the health care provider. c. Take the patient's pulse and blood pressure. d. Determine the patient's SpO2 with an oximeter.

a. Raise the head of the bed.

A patient with a recent diagnosis of heart failure has been prescribed furosemide (Lasix) in an effort to physiologically do what for the patient? a. Reduce preload. b. Decrease afterload. c. Increase contractility. d. Promote vasodilation.

a. Reduce preload. Diuretics such as furosemide are used in the treatment of HF to mobilize edematous fluid, reduce pulmonary venous pressure, and reduce preload. They do not directly influence afterload, contractility, or vessel tone.

Following assessment of a patient with pneumonia, the nurse identifies a nursing diagnosis of impaired gas exchange based on which finding? a. SpO2 of 86% c. Temperature of 101.4ºF (38.6ºC) b. Crackles in both lower lobes d. Production of greenish purulent sputum

a. SpO2 of 86%

While the nurse is feeding a patient, the patient appears to choke on the food. Which symptoms indicate to the nurse that the patient has a partial airway obstruction? (select all that apply) a. Stridor b. Cyanosis c. Wheezing d. Bradycardia e. Rapid respiratory rate

a. Stridor b. Cyanosis c. Wheezing

Delegation Decision: In planning the care for a patient with a tracheostomy who has been stable and is to be discharged later in the day, the RN may delegate which interventions to the licensed practical nurse (LPN) ? (select all that apply) a. Suction the tracheostomy. b. Provide tracheostomy care. c. Determine the need for suctioning. d. Assess the patient's swallowing ability. e. Teach the patient about home tracheostomy care.

a. Suction the tracheostomy. b. Provide tracheostomy care. c. Determine the need for suctioning.

During preoperative teaching for the patient scheduled for a total laryngectomy, what should the nurse include? a. The postoperative use of nonverbal communication techniques b. Techniques that will be used to alleviate a dry mouth and prevent stomatitis c. The need for frequent, vigorous coughing in the first 24 hours postoperatively d. Self-help groups and community resources for patients with cancer of the larynx

a. The postoperative use of nonverbal communication techniques

During a health promotion program, why should the nurse plan to target women in a discussion of lung cancer prevention? (select all that apply) a. Women develop lung cancer at a younger age than men. b. More women die of lung cancer than die from breast cancer. c. Women have a worse prognosis from lung cancer than do men. d. Women are more likely to develop small cell carcinoma than men. e. Nonsmoking women are at greater risk for developing lung cancer than men.

a. Women develop lung cancer at a younger age than men. b. More women die of lung cancer than die from breast cancer. d. Women are more likely to develop small cell carcinoma than men. e. Nonsmoking women are at greater risk for developing lung cancer than men.

The nurse recognizes that primary manifestations of systolic failure include: a. decreased EF and increased PAWP b. decreased PAWP and increased EF. c. decreased pulmonary hypertension associated with normal EF d. decreased afterload and decreased left ventricular end-diastolic pressure

a. decreased EF and increased PAWP Rationale: Systolic heart failure results in systolic failure in the left ventricle (LV). The LV loses its ability to generate enough pressure to eject blood forward through the aorta. This results in increased pulmonary artery wedge pressure (PAWP). The hallmark of systolic failure is a decrease in the left ventricular ejection fraction (EF).

The patient has been diagnosed with an early vocal cord malignancy. The nurse explains that usual treatment includes a. radiation therapy that preserves the quality of the voice. b. a hemilaryngectomy that prevents the need for a tracheostomy. c. a radical neck dissection that removes possible sites of metastasis. d. a total laryngectomy to prevent development of second primary cancers.

a. radiation therapy that preserves the quality of the voice.

Patients with a heart transplantation are at risk for which complications in the first year after transplantation (select all that apply)? a. cancer b. infection c. rejection d. vasculopathy e. sudden cardiac death

b, c, & e Rationale: A variety of complications can occur after heart transplantation. In the first year after transplantation, the major causes of death are acute rejection and infection. Heart transplant recipients also are at risk for sudden cardiac death. Later, malignancy (especially lymphoma) and cardiac vasculopathy (accelerated CAD) are major causes of death.

A patient with advanced lung cancer refuses pain medication, saying, "I deserve everything this cancer can give me." What is the nurse's best response to this patient? a. "Would talking to a counselor help you?" b. "Can you tell me what the pain means to you?" c. "Are you using the pain as a punishment for your smoking?" d. "Pain control will help you to deal more effectively with your feelings."

b. "Can you tell me what the pain means to you?"

What is the most normal functioning method of speech restoration in the patient with a total laryngectomy? a. Esophageal speech b. A transesophageal puncture c. An electrolarynx held to the neck d. An electrolarynx placed in the mouth

b. A transesophageal puncture

Priority Decision: A patient diagnosed with class 3 TB 1 week ago is admitted to the hospital with symptoms of chest pain and coughing. What nursing action has the highest priority? a. Administering the patient's antitubercular drugs b. Admitting the patient to an airborne infection isolation room c. Preparing the patient's room with suction equipment and extra linens d. Placing the patient in an intensive care unit where he can be closely monitored

b. Admitting the patient to an airborne infection isolation room

A patient with a diagnosis of heart failure has been started on a nitroglycerin patch by his primary care provider. What should this patient be taught to avoid? a. High-potassium foods b. Drugs to treat erectile dysfunction c. Nonsteroidal antiinflammatory drugs d. Over-the-counter H2 -receptor blockers

b. Drugs to treat erectile dysfunction The use of erectile drugs concurrent with nitrates creates a risk of severe hypotension and possibly death. High-potassium foods, NSAIDs, and H2-receptor blockers do not pose a risk in combination with nitrates.

When obtaining a health history from a patient suspected of having early TB, what manifestations should the nurse ask the patient about? a. Chest pain, hemoptysis, and weight loss b. Fatigue, low-grade fever, and night sweats c. Cough with purulent mucus and fever with chills d. Pleuritic pain, nonproductive cough, and temperature elevation at night

b. Fatigue, low-grade fever, and night sweats

When obtaining a health history from a patient with possible cancer of the mouth, what would the nurse expect the patient to report? a. Long-term denture use b. Heavy tobacco and/or alcohol use c. Persistent swelling of the neck and face d. Chronic herpes simplex infections of the mouth and lips

b. Heavy tobacco and/or alcohol use

What should the nurse include in discharge teaching for the patient with a total laryngectomy? a. How to use esophageal speech to communicate b. How to use a mirror to suction the tracheostomy c. The necessity of never covering the laryngectomy stoma d. The need to use baths instead of showers for personal hygiene

b. How to use a mirror to suction the tracheostomy

What should the nurse recognize as an indication for the use of dopamine (Intropin) in the care of a patient with heart failure? a. Acute anxiety b. Hypotension and tachycardia c. Peripheral edema and weight gain d. Paroxysmal nocturnal dyspnea (PND)

b. Hypotension and tachycardia Dopamine is a β-adrenergic agonist whose inotropic action is used for treatment of severe heart failure accompanied by hemodynamic instability. Such a state may be indicated by tachycardia accompanied by hypotension. PND, anxiety, edema, and weight gain are common signs and symptoms of heart failure, but these do not necessarily warrant the use of dopamine.

What does the nurse teach the patient with intermittent allergic rhinitis is the most effective way to decrease allergic symptoms? a. Undergo weekly immunotherapy. b. Identify and avoid triggers of the allergic reaction. c. Use cromolyn nasal spray prophylactically year-round. d. Use over-the-counter antihistamines and decongestants during an acute attack.

b. Identify and avoid triggers of the allergic reaction.

A patient with pneumonia has a nursing diagnosis of ineffective airway clearance related to pain, fatigue, and thick secretions. What is an expected outcome for this patient? a. SpO2 is 90% b. Lungs clear to auscultation c. Patient tolerates walking in hallway d. Patient takes three or four shallow breaths before coughing to minimize pain

b. Lungs clear to auscultation

What is included in the nursing care of the patient with a cuffed tracheostomy tube? a. Change the tube every 3 days. b. Monitor cuff pressure every 8 hours. c. Perform mouth care every 12 hours. d. Assess arterial blood gases every 8 hours.

b. Monitor cuff pressure every 8 hours.

During an annual health assessment of a 65-year-old patient at the clinic, the patient tells the nurse he had the pneumonia vaccine when he was age 58. What should the nurse advise him about the best way for him to prevent pneumonia? a. Seek medical care and antibiotic therapy for all upper respiratory infections b. Obtain the pneumococcal vaccine this year with an annual influenza vaccine c. Obtain the pneumococcal vaccine if he is exposed to individuals with pneumonia d. Obtain only the influenza vaccine every year because he has immunity to the pneumococcus

b. Obtain the pneumococcal vaccine this year with an annual influenza vaccine

The microorganisms Pneumocystis jiroveci (PCP) and cytomegalovirus (CMV) are associated with which type of pneumonia? a. Bronchial pneumonia b. Opportunistic pneumonia c. Hospital-associated pneumonia d. Community-acquired pneumonia

b. Opportunistic pneumonia

What was the resurgence in tuberculosis (TB) resulting from the emergence of multidrug-resistant strains of Mycobacterium tuberculosis related to? a. A lack of effective means to diagnose TB b. Poor compliance with drug therapy in patients with TB c. Indiscriminate use of antitubercular drugs in treatment of other infections d. Increased population of immunosuppressed individuals with acquired immunodeficiency syndrome (AIDS)

b. Poor compliance with drug therapy in patients with TB

During care of a patient with a cuffed tracheostomy, the nurse notes that the tracheostomy tube has an inner cannula. To care for the tracheostomy appropriately, what should the nurse do? a. Deflate the cuff, then remove and suction the inner cannula. b. Remove the inner cannula and replace it per institutional guidelines. c. Remove the inner cannula if the patient shows signs of airway obstruction. d. Keep the inner cannula in place at all times to prevent dislodging the tracheostomy tube.

b. Remove the inner cannula and replace it per institutional guidelines.

While caring for a patient with idiopathic pulmonary arterial hypertension (IPAH), the nurse observes that the patient has exertional dyspnea and chest pain in addition to fatigue. What are these symptoms related to? a. Decreased left ventricular output b. Right 3 c. Increased systemic arterial blood pressure

b. Right 3

To determine whether a tension pneumothorax is developing in a patient with chest trauma, for what does the nurse assess the patient? a. Dull percussion sounds on the injured side b. Severe respiratory distress and tracheal deviation c. Muffled and distant heart sounds with decreasing blood pressure d. Decreased movement and diminished breath sounds on the affected side

b. Severe respiratory distress and tracheal deviation

Priority Decision: To reduce the risk for most occupational lung diseases, what is the most important measure the occupational nurse should promote? a. Maintaining smoke-free work environments for all employees. b. Using masks and effective ventilation systems to reduce exposure to irritants. c. Inspection and monitoring of workplaces by national occupational safety agencies. d. Requiring periodic chest x-rays and pulmonary function tests for exposed employees.

b. Using masks and effective ventilation systems to reduce exposure to irritants.

The nurse is preparing to administer digoxin to a patient with heart failure. In preparation, laboratory results are reviewed with the following findings: sodium 139 mEq/L, potassium 5.6 mEq/L, chloride 103 mEq/L, and glucose 106 mg/dL. What should the nurse do next? a. Withhold the daily dose until the following day. b. Withhold the dose and report the potassium level. c. Give the digoxin with a salty snack, such as crackers. d. Give the digoxin with extra fluids to dilute the sodium level.

b. Withhold the dose and report the potassium level. The normal potassium level is 3.5 to 5.0 mEq/L. The patient is hyperkalemic, which makes the patient more prone to digoxin toxicity. For this reason, the nurse should withhold the dose and report the potassium level. The physician may order the digoxin to be given once the potassium level has been treated and decreases to within normal range.

What is an advantage of a tracheostomy over an endotracheal (ET) tube for long-term management of an upper airway obstruction? a. A tracheostomy is safer to perform in an emergency. b. An ET tube has a higher risk of tracheal pressure necrosis. c. A tracheostomy tube allows for more comfort and mobility. d. An ET tube is more likely to lead to lower respiratory tract infection.

c. A tracheostomy tube allows for more comfort and mobility.

A patient with a lung mass found on chest x-ray is undergoing further testing. The nurse explains that a diagnosis of lung cancer can be confirmed using which diagnostic test? a. Lung tomograms b. Pulmonary angiography c. Biopsy done via bronchoscopy d. Computed tomography (CT) scans

c. Biopsy done via bronchoscopy

What is the priority assessment by the nurse caring for a patient receiving IV nesiritide (Natrecor) to treat heart failure? a. Urine output b. Lung sounds c. Blood pressure d. Respiratory rate

c. Blood pressure Although all identified assessments are appropriate for a patient receiving IV nesiritide, the priority assessment would be monitoring for hypotension, the main adverse effect of nesiritide.

Following a supraglottic laryngectomy, the patient is taught how to use the supraglottic swallow to minimize the risk of aspiration. In teaching the patient about this technique, what should the nurse instruct the patient to do? a. Perform Valsalva maneuver immediately after swallowing. b. Breathe between each Valsalva maneuver and cough sequence. c. Cough after swallowing to remove food from the top of the vocal cords. d. Practice swallowing thin, watery fluids before attempting to swallow solid foods.

c. Cough after swallowing to remove food from the top of the vocal cords.

A patient with an acute pharyngitis is seen at the clinic with fever and severe throat pain that affects swallowing. On inspection, the throat is reddened and edematous with patchy yellow exudates. The nurse anticipates that collaborative management will include: a. treatment with antibiotics. b. treatment with antifungal agents. c. a throat culture or rapid strep antigen test. d. treatment with medication only if the pharyngitis does not resolve in 3 to 4 days.

c. a throat culture or rapid strep antigen test.

A compensatory mechanism involved in HF that leads to inappropriate fluid retention and additional workload of the heart is: a. ventricular dilation b. ventricular hypertrophy c. neurohormonal response d. sympathetic nervous system activation

c. neurohormonal response Rationale: The following mechanisms in heart failure lead to inappropriate fluid retention and additional workload of the heart: activation of the renin-angiotensin-aldosterone system (RAAS) cascade and release of antidiuretic hormone from the posterior pituitary gland in response to low cerebral perfusion pressure that results from low cardiac output.

An unlicensed assistive personnel (UAP) is taking care of a patient with a chest tube. The nurse should intervene when she observes the UAP a. looping the drainage tubing on the bed. b. securing the drainage container in an upright position. c. stripping or milking the chest tube to promote drainage. d. reminding the patient to cough and deep breathe every 2 hours.

c. stripping or milking the chest tube to promote drainage.

Muffled and distant heart sounds indicate a

cardiac tamponade.

A patient who cannot remember to take the medication usually will not remember to

come to the clinic daily or will find it too inconvenient.

A patient with a 40 pack-year smoking history has recently stopped smoking because of the fear of developing lung cancer. The patient asks the nurse what he can do to learn about whether he develops lung cancer. What is the best response from the nurse? a. "You should get a chest x-ray every 6 months to screen for any new growths." b. "It would be very rare for you to develop lung cancer now that you have stopped smoking." c. "You should monitor for any persistent cough, wheezing, or difficulty breathing, which could indicate tumor growth." d. "Screening measures for lung cancers are controversial, but we can discuss the advantages and disadvantages of various measures."

d. "Screening measures for lung cancers are controversial, but we can discuss the advantages and disadvantages of various measures."

The nurse is administering a dose of digoxin (Lanoxin) to a patient with heart failure (HF). The nurse would become concerned with the possibility of digitalis toxicity if the patient reported which symptom(s)? a. Muscle aches b. Constipation c. Pounding headache d. Anorexia and nausea

d. Anorexia and nausea Anorexia, nausea, vomiting, blurred or yellow vision, and cardiac dysrhythmias are all signs of digitalis toxicity. The nurse would become concerned and notify the health care provider if the patient exhibited any of these symptoms.

A patient with active TB continues to have positive sputum cultures after 6 months of treatment. She says she cannot remember to take the medication all the time. What is the best action for the nurse to take? a. Schedule the patient to come to the clinic every day to take the medication. b. Have a patient who has recovered from TB tell the patient about his successful treatment. c. Schedule more teaching sessions so the patient will understand the risks of noncompliance. d. Arrange for directly observed therapy by a responsible family member or a public health nurse.

d. Arrange for directly observed therapy by a responsible family member or a public health nurse.

A stable patient with acute decompensated heart failure (ADHF) suddenly becomes dyspneic. Before positioning the patient on the bedside, what should the nurse assess first? a. Urine output b. Heart rhythm c. Breath sounds d. Blood pressure

d. Blood pressure The nurse should evaluate the blood pressure before dangling the patient on the bedside because the blood pressure can decrease as blood pools in the periphery and preload decreases. If the patient's blood pressure is low or marginal, the nurse should put the patient in the semi-Fowler's position and use other measures to improve gas exchange.

Beyond the first year after a heart transplant, the nurse knows that what is a major cause of death? a. Infection b. Acute rejection c. Immunosuppression d. Cardiac vasculopathy

d. Cardiac vasculopathy Beyond the first year after a heart transplant, malignancy (especially lymphoma) and cardiac vasculopathy (accelerated CAD) are the major causes of death. During the first year after transplant, infection and acute rejection are the major causes of death. Immunosuppressive therapy will be used for posttransplant management to prevent rejection and increases the patient's risk of an infection.

A male patient with a long-standing history of heart failure has recently qualified for hospice care. What measure should the nurse now prioritize when providing care for this patient? a. Taper the patient off his current medications. b. Continue education for the patient and his family. c. Pursue experimental therapies or surgical options. d. Choose interventions to promote comfort and prevent suffering.

d. Choose interventions to promote comfort and prevent suffering. The central focus of hospice care is the promotion of comfort and the prevention of suffering. Patient education should continue, but providing comfort is paramount. Medications should be continued unless they are not tolerated. Experimental therapies and surgeries are not commonly used in the care of hospice patients.

During assessment of the patient with a viral upper respiratory infection, the nurse recognizes that antibiotics may be indicated based on what finding? a. Cough and sore throat b. Copious nasal discharge c. Temperature of 100°F (38°C) d. Dyspnea and severe sinus pain

d. Dyspnea and severe sinus pain

What is the initial antibiotic treatment for pneumonia based on? a. The severity of symptoms b. The presence of characteristic leukocytes c. Gram stains and cultures of sputum specimens d. History and physical examination and characteristic chest x-ray findings

d. History and physical examination and characteristic chest x-ray findings

The use of radiology, computed tomography (CT), and sputum cytology has been shown to

detect lung cancer at earlier stages.

If drug compliance is questionable, follow-up of patients can be made by

directly observed therapy by a public health nurse or a responsible family member

What are the characteristics of a fenestrated tracheostomy tube? (select all that apply) a. The cuff passively fills with air. b. Cuff pressure monitoring is not required. c. It has two tubings with one opening just above the cuff. d. Patient can speak with an attached air source with the cuff inflated. e. Airway obstruction is likely if the exact steps are not followed to produce speech. f. Airflow around the tube and through the window allows speech when the cuff is deflated and the plug is inserted.

e. Airway obstruction is likely if the exact steps are not followed to produce speech. f. Airflow around the tube and through the window allows speech when the cuff is deflated and the plug is inserted.

The INJECTED TRIVALENT INACTIVATED influenza vaccine is recommended for a. individuals 6 months of age and older b. those at increased risk for influenza-related complications c. people with chronic medical conditions or those who are immunocompromised d. residents of long-term care facilities e. health care workers, and providers of care to at-risk persons f. All of the above

f. All of the above

TB usually develops insidiously with

fatigue, malaise, anorexia, low-grade fevers, and night sweats.

Chronic infections are not known to be risk factors, although cancers in patients younger than age 50 have been associated with

human papillomavirus (HPV) infection.

Although chest x-rays, lung tomograms, CT scans, MRI, and positron emission tomography (PET) can identify tumors and masses, a definitive diagnosis of a lung cancer requires

identification of malignant cells in either sputum specimens or biopsies.

Transesophageal puncture provides the most

normal voice reproduction

Confusion possibly related to hypoxia may be the only finding in

older adults.

A tension pneumothorax causes many of the same manifestations as

other types of pneumothoraces

Dry mouth and stomatitis result from

radiation therapy

Notification of the public health department is

required.

patients must be 55 to 74 years old, current smokers with at least a 30 pack-year smoking history or former smokers who quit within the past 15 years, have no history of lung cancer, and not be on home oxygen To be considered for

screening for lung cancer,

Severe respiratory distress from collapse of the entire lung with movement of the mediastinal structures and trachea to the unaffected side is present in a

tension pneumothorax

Cardiac monitoring and observation will need to be done with

the patient in isolation.

A patient who has a smoking history always has an increased risk for lung cancer compared with an individual who has never smoked but

the risk decreases as the period of nonsmoking increases.

If the sputum specimen cannot be obtained rapidly, the chest x-ray will be done to assess

the typical pattern characteristic of the infecting organism.

The LIVE ATTENUATED influenza vaccine is given intranasally and is recommended for all healthy people between the ages of 2 and 49 but NOT for

those at increased risk of complications or health care providers

Low-dose spiral CT scanning has been shown to decrease lung cancer mortality compared with

those who had chest x-rays.

Rifabutin (Mycobutin) and levofloxacin (Levaquin) may be used if the patient develops

toxicity to the primary drugs.

Esophageal speech involves

trapping air in the esophagus and releasing it to form sound Only 10% of patients can develop fluent speech

Flail chest may occur when two or more ribs are fractured, causing a(n)

unstable segment.


Set pelajaran terkait

ACG 302 Exam 2 Concept Questions

View Set

Chapter 20: Taxation and the Public Budget

View Set

Chapter 4: Savings Plans and Payment Accounts

View Set

the parts of the brain; unit 3, lesson 3

View Set

Chapter 57; Coordinating Care for Patients with Stomach Disorders

View Set

Nutrition and Wellness Review Unit-2

View Set

operations management final review

View Set

Exam 2 Math Review (5,6,7) FINA 3332

View Set